rest of the peds class

¡Supera tus tareas y exámenes ahora con Quizwiz!

The nurse is caring for an infant with developmental dysplasia of the hip. Which clinical manifestations should the nurse expect to observe (Select all that apply)? a. Positive Ortolani sign b. Unequal gluteal folds c. Negative Babinskis sign d. Trendelenburgs sign e. Telescoping of the affected limb f. Lordosis

ANS: A, B A positive Ortolani sign and unequal gluteal folds are clinical manifestations of developmental dysplasia of the hip seen from birth to 2 to 3 months. Negative Babinskis sign, Trendelenburgs sign, telescoping of the affected limb, and lordosis are not clinical manifestations of developmental dysplasia of the hip

A school-age child is diagnosed with systemic lupus erythematosus (SLE). The nurse should plan to implement which interventions for this child (Select all that apply)? a. Instructions to avoid exposure to sunlight b. Teaching about body changes associated with SLE c. Preparation for home schooling d. Restricted activity

ANS: A, B Key issues for a child with SLE include therapy compliance; body-image problems associated with rash, hair loss, and steroid therapy; school attendance; vocational activities; social relationships; sexual activity; and pregnancy. Specific instructions for avoiding exposure to the sun and ultraviolet B light, such as using sunscreens, wearing sun-resistant clothing, and altering outdoor activities, must be provided with great sensitivity to ensure compliance while minimizing the associated feeling of being different from peers. The child should continue school attendance in order to gain interaction with peers and activity should not be restricted, but promoted.

The nurse in the neonatal intensive care unit is caring for an infant with myelomeningocele scheduled for surgical repair in the morning. Which early signs of infection should the nurse monitor on this infant (Select all that apply)? a. Temperature instability b. Irritability c. Lethargy d. Bradycardia e. Hypertension

ANS: A, B, C The nurse should observe an infant with unrepaired myelomeningocele for early signs of infection, such as temperature instability (axillary), irritability, and lethargy. Bradycardia and hypertension are not early signs of infection in infants

The nurse is conducting discharge teaching with parents of a preschool child with myelomeningocele, repaired at birth, who is being discharged from the hospital after a urinary tract infection (UTI). Which should the nurse include in the discharge instructions related to management of the childs genitourinary function (Select all that apply)? a. Continue to perform the clean intermittent catheterizations (CIC) at home. b. Administer the oxybutynin chloride (Ditropan) as prescribed. c. Reduce fluid intake in the afternoon and evening hours. d. Monitor for signs of a recurrent UTI. e. Administer furosemide (Lasix) as prescribed.

ANS: A, B, D Discharge teaching to prevent renal complications in a child with myelomeningocele include: (1) regular urologic care with prompt and vigorous treatment of infections; (2) a method of regular emptying of the bladder, such as clean intermittent catheterization (CIC) taught to and performed by parents and self- catheterization taught to children; and (3) medications to improve bladder storage and continence, such as oxybutynin chloride (Ditropan) and tolterodine (Detrol). Fluids should not be limited, and Lasix is not used to improve renal function for children with myelomeningocele

Which assessment findings should the nurse note in a school-age child with Duchennes muscular dystrophy (DMD) (Select all that apply)? a. Lordosis b. Gowers sign c. Kyphosis d. Scoliosis e. Waddling gait

ANS: A, B, E Difficulties in running, riding a bicycle, and climbing stairs are usually the first symptoms noted in DMD. Typically, affected boys have a waddling gait and lordosis, fall frequently, and develop a characteristic manner of rising from a squatting or sitting position on the floor (Gowers sign). Lordosis occurs as a result of weakened pelvic muscles, and the waddling gait is a result of weakness in the gluteus medius and maximus muscles. Kyphosis and scoliosis are not assessment findings with DMD

A 14-year-old girl is in the intensive care unit after a spinal cord injury 2 days ago. Which nursing care interventions are needed for this child (Select all that apply)? a. Monitoring and maintaining systemic blood pressure. b. Administering corticosteroids. c. Minimizing environmental stimuli. d. Discussing long-term care issues with the family. e. Monitoring for respiratory complications.

ANS: A, B, E Spinal cord injury patients are physiologically labile, and close monitoring of blood pressure and respirations is required. They may be unstable for the first few weeks after the injury. Corticosteroids are administered to minimize the inflammation present with the injury. Minimizing environmental stimuli and discussing longterm care issues with the family do not apply to providing care for this patient.

A nurse should expect which cerebrospinal fluid (CSF) laboratory results on a child diagnosed with bacterial meningitis (Select all that apply)? a. Elevated white blood cell (WBC) count b. Decreased protein c. Decreased glucose d. Cloudy in color e. Increase in red blood cells (RBCs)

ANS: A, C, D Test Bank - Maternal Child Nursing Care by Perry (6th Edition, 2017) 791 The CSF laboratory results for bacterial meningitis include elevated WBC counts, cloudy or milky in color, and decreased glucose. The protein is elevated and there should be no RBCs present. RBCs are present when the tap was traumatic.

The treatment of brain tumors in children consists of which therapies(Select all that apply)? a. Surgery b. Bone marrow transplantation c. Chemotherapy d. Stem cell transplantation e. Radiation f. Myelography

ANS: A, C, E Treatment for brain tumors in children may consist of surgery, chemotherapy, and radiotherapy alone or in combination. Bone marrow, stem cell, and myelographuy are transplantation therapies are not used to treat brain tumors in children

An infant with hydrocephalus is hospitalized for surgical placement of a ventriculoperitoneal shunt. Which interventions should be included in the childs postoperative care (Select all that apply)? a. Observe closely for signs of infection. b. Pump the shunt reservoir to maintain patency. c. Administer sedation to decrease irritability. d. Maintain Trendelenburg position to decrease pressure on the shunt. e. Maintain an accurate record of intake and output. f. Monitor for abdominal distention.

ANS: A, E, F Infection is a major complication of ventriculoperitoneal shunts. Observation for signs of infection is a priority nursing intervention. Intake and output should be measured carefully. Abdominal distention could be a sign of peritonitis or a postoperative ileus. Pumping the shunt reservoir, administering sedation, and maintaining Trendelenburg position are not interventions associated with this condition

A school-age child has sustained a head injury and multiple fractures after being thrown from a horse. The childs level of consciousness is variable. The parents tell the nurse that they think their child is in pain because of periodic crying and restlessness. The most appropriate nursing action is to: Test Bank - Maternal Child Nursing Care by Perry (6th Edition, 2017) 779 a. Discuss with parents the childs previous experiences with pain. b. Discuss with practitioner what analgesia can be safely administered. c. Explain that analgesia is contraindicated with a head injury. d. Explain that analgesia is unnecessary when child is not fully awake and alert.

ANS: B A key nursing role is to provide sedation and analgesia for the child. Consultation with the appropriate practitioner is necessary to avoid conflict between the necessity to monitor the childs neurologic status and to promote comfort and relieve anxiety. Gathering information about the childs previous experiences with pain should be obtained as part of the assessment, but because of the severity of injury, analgesia should be provided as soon as possible. Analgesia can be used safely in individuals who have sustained head injuries and can decrease anxiety and resultant increased intracranial pressure.

Which statement best describes a subdural hematoma? a. Bleeding occurs between the dura and the skull. b. Bleeding occurs between the dura and the cerebrum. c. Bleeding is generally arterial, and brain compression occurs rapidly. d. The hematoma commonly occurs in the parietotemporal region

ANS: B A subdural hematoma is bleeding that occurs between the dura and the cerebrum as a result of a rupture of cortical veins that bridge the subdural space. An epidural hemorrhage occurs between the dura and the skull, is usually arterial with rapid brain concussion, and occurs most often in the parietotemporal region

An advantage to using a fiberglass cast instead of a plaster cast is that a fiberglass cast: a. Is less expensive. b. Dries rapidly. c. Molds closely to body parts. d. Has a smooth exterior.

ANS: B A synthetic casting material dries in 5 to 30 minutes as compared with a plaster cast, which takes 10 to 72 hours to dry. Synthetic casts are more expensive. Plaster casts mold closer to body parts. Synthetic casts have a rough exterior, which may scratch surfaces.

The nurse is caring for an infant with myelomeningocele scheduled for surgical closure in the morning. Which interventions should the nurse plan for the care of the myelomeningocele sac? a. Open to air b. Covered with a sterile, moist, nonadherent dressing c. Reinforcement of the original dressing if drainage noted d. A diaper secured over the dressing

ANS: B Before surgical closure, the myelomeningocele is prevented from drying by the application of a sterile, moist, nonadherent dressing over the defect. The moistening solution is usually sterile normal saline. Dressings are changed frequently (every 2 to 4 hours), and the sac is closely inspected for leaks, abrasions, irritation, and any signs of infection. The sac must be carefully cleansed if it becomes soiled or contaminated. The original dressing would not be reinforced but changed as needed. A diaper is not placed over the dressing because stool contamination can occur.

Kristin, age 10 years, sustained a fracture in the epiphyseal plate of her right fibula when she fell off of a tree. When discussing this injury with her parents, the nurse should consider which statement? a. Healing is usually delayed in this type of fracture. b. Growth can be affected by this type of fracture. c. This is an unusual fracture site in young children. d. This type of fracture is inconsistent with a fall.

ANS: B Detection of epiphyseal injuries is sometimes difficult, but fractures involving the epiphysis or epiphyseal plate present special problems in determining whether bone growth will be affected. Healing of epiphyseal injuries is usually prompt. The epiphysis is the weakest point of the long bones. This is a frequent site of damage during trauma

The nurse is preparing a school-age child for a computed tomography (CT) scan to assess cerebral function. When preparing the child for the scan, which statement should the nurse include? a. Pain medication will be given. Test Bank - Maternal Child Nursing Care by Perry (6th Edition, 2017) 775 b. The scan will not hurt. c. You will be able to move once the equipment is in place. d. Unfortunately no one can remain in the room with you during the test

ANS: B For CT scans, the child will not be allowed to move and must be immobilized. It is important to emphasize to the child that at no time is the procedure painful. Pain medication is not required; however, sedation is sometimes necessary. Someone is able to remain with the child during the procedure.

The nurse is caring for a 4-year-old child immobilized by a fractured hip. Which complications should the nurse monitor? a. Hypocalcemia b. Decreased metabolic rate c. Positive nitrogen balance d. Increased production of stress hormones

ANS: B Immobilization causes a decreased metabolic rate with slowing of all systems and a decreased food intake, leads to hypercalcemia, and causes a negative nitrogen balance secondary to muscle atrophy. A decreased production of stress hormones occurs with decreased physical and emotional coping capacity.

The nurse is caring for a neonate born with a myelomeningocele. Surgery to repair the defect is scheduled the next day. The most appropriate way to position and feed this neonate is to place him: a. Prone and tube feed. b. Prone, turn head to side, and nipple feed. c. Supine in infant carrier and nipple feed. d. Supine, with defect supported with rolled blankets, and nipple feed

ANS: B In the prone position, feeding is a problem. The infants head is turned to one side for feeding. If the child is able to nipple feed, no indication is present for tube feeding. Before surgery, the infant is kept in the prone position to minimize tension on the sac and risk of trauma.

How much folic acid is recommended for women of childbearing age? a. 0.1 mg b. 0.4 mg c. 1.5 mg d. 2 mg

ANS: B It has been estimated that a daily intake of 0.4 mg of folic acid in women of childbearing age will prevent 50% to 70% of cases of neural tube defects. A dose of 0.1 mg is too low, and 1.5 mg and 2 mg are not recommended dosages of folic acid.

When taking the history of a child hospitalized with Reyes syndrome, the nurse should not be surprised that a week ago the child had recovered from: a. Measles. c. Meningitis. b. Varicella. d. Hepatitis.

ANS: B Most cases of Reyes syndrome follow a common viral illness such as varicella or influenza. Measles, meningitis, and hepatitis are not associated with Reyes syndrome

Which interaction is part of the discharge plan for a school-age child with osteomyelitis who is receiving home antibiotic therapy? a. Instructions for a low-calorie diet b. Arrangements for tutoring and schoolwork c. Instructions for a high-fat, low-protein diet d. Instructions for the parent to return the child to team sports immediately

ANS: B Promoting optimal growth and development in the school-age child is important. It is important to continue schoolwork and arrange for tutoring if indicated. The child with osteomyelitis should be on a high-calorie, high-protein diet. The child with osteomyelitis may need time for the bone to heal before returning to full activities.

. Which finding should cause the nurse to suspect a diagnosis of spastic cerebral palsy? a. Tremulous movements at rest and with activity b. Sudden jerking movement caused by stimuli c. Writhing, uncontrolled, involuntary movements d. Clumsy, uncoordinated movements

ANS: B Spastic cerebral palsy, the most common type of cerebral palsy, will manifest with hypertonicity and increased deep tendon reflexes. The childs muscles are very tight and any stimuli may cause a sudden jerking movement. Tremulous movements are characteristic of rigid/tremor/atonic cerebral palsy. Slow, writhing, uncontrolled, involuntary movements occur with athetoid or dyskinetic cerebral palsy. Clumsy movements, loss of coordination, equilibrium, and kinesthetic sense occur in ataxic cerebral palsy.

Which term is used when a patient remains in a deep sleep, responsive only to vigorous and repeated stimulation? a. Coma c. Obtundation b. Stupor d. Persistent vegetative state

ANS: B Stupor exists when the child remains in a deep sleep, responsive only to vigorous and repeated stimulation. Coma is the state in which no motor or verbal response occurs to noxious (painful) stimuli. Obtundation describes a level of consciousness in which the child can be aroused with stimulation. Persistent vegetative state describes the permanent loss of function of the cerebral cortex.

The Glasgow Coma Scale consists of an assessment of: a. Pupil reactivity and motor response. b. Eye opening and verbal and motor responses. c. Level of consciousness and verbal response. d. Intracranial pressure (ICP) and level of consciousness.

ANS: B Test Bank - Maternal Child Nursing Care by Perry (6th Edition, 2017) 774 The Glasgow Coma Scale assesses eye opening and verbal and motor responses. Pupil reactivity is not a part of the Glasgow Coma Scale but is included in the pediatric coma scale. Level of consciousness and ICP are not part of the Glasgow Coma Scale.

The nurse has received report on four children. Which child should the nurse assess first? a. A school-age child in a coma with stable vital signs b. A preschool child with a head injury and decreasing level of consciousness c. An adolescent admitted after a motor vehicle accident who is oriented to person and place d. A toddler in a persistent vegetative state with a low-grade fever

ANS: B The nurse should assess the child with a head injury and decreasing level of consciousness (LOC) first. Assessment of LOC remains the earliest indicator of improvement or deterioration in neurologic status. The next child the nurse should assess is a toddler in a persistent vegetative state with a low-grade fever. The school-age child in a coma with stable vital signs and the adolescent admitted to the hospital who is oriented to his or her surroundings would be of least worry to the nurse.

The nurse is closely monitoring a child who is unconscious after a fall and notices that the child suddenly has a fixed and dilated pupil. The nurse should interpret this as: a. Eye trauma. c. Severe brainstem damage. b. Neurosurgical emergency. d. Indication of brain death.

ANS: B The sudden appearance of a fixed and dilated pupil(s) is a neurosurgical emergency. The nurse should immediately report this finding. Although a dilated pupil may be associated with eye trauma, this child has experienced a neurologic insult. Pinpoint pupils or fixed, bilateral pupils for more than 5 minutes are indicative of brainstem damage. The unilateral fixed and dilated pupil is suggestive of damage on the same side of the brain. One fixed and dilated pupil is not suggestive of brain death.

An appropriate nursing intervention when caring for a child in traction is to: a. Remove adhesive traction straps daily to prevent skin breakdown. b. Assess for tightness, weakness, or contractures in uninvolved joints and muscles. c. Provide active range-of-motion exercises to affected extremity 3 times a day. d. Keep child in one position to maintain good alignment.

ANS: B Traction places stress on the affected bone, joint, and muscles. The nurse must assess for tightness, weakness, or contractures developing in the uninvolved joints and muscles. The adhesive straps should be released/replaced only when absolutely necessary. Active, passive, or active with resistance exercises should be carried out for the unaffected extremity only. Movement is expected with children. Each time the child moves, the nurse should check to ensure that proper alignment is maintained.

The nurse is admitting a school-age child with suspected Guillain-Barr syndrome (GBS). Which nursing intervention is a priority in the care for this child? a. Monitoring intake and output b. Assessing respiratory efforts c. Placing on a telemetry monitor d. Obtaining laboratory studies

ANS: B Treatment of GBS is primarily supportive. In the acute phase, patients are hospitalized because respiratory and pharyngeal involvement may require assisted ventilation, sometimes with a temporary tracheotomy. Treatment modalities include aggressive ventilatory support in the event of respiratory compromise, administration of intravenous immunoglobulin (IVIG), and sometimes steroids; plasmapheresis and immunosuppressive drugs may also be used. Monitoring intake and output, telemetry monitoring, and obtaining laboratory studies may be part of the plan of care but are not the priority.

The vector reservoir for agents causing viral encephalitis in the United States is: a. Tarantula spiders. c. Carnivorous wild animals. b. Mosquitoes and ticks. d. Domestic and wild animals.

ANS: B Viral encephalitis, not attributable to a childhood viral disease, is usually transmitted by mosquitoes and ticks. The vector reservoir for most agents pathogenic for humans and detected in the United States are mosquitoes and ticks; therefore, most cases of encephalitis appear during the hot summer months. Tarantulas, carnivorous wild animals, and domestic animals are not reservoirs for the agents that cause viral encephalitis.

A clinic nurse is conducting a staff in-service for other clinic staff regarding the signs and symptoms of a rhabdomyosarcoma tumor. Which should be included in the teaching session (Select all that apply)? a. Bone fractures b. Abdominal mass c. Sore throat and ear pain d. Headache e. Ecchymosis of conjunctiva

ANS: B, C, E The initial signs and symptoms of rhabdomyosarcoma tumors are related to the site of the tumor and compression of adjacent organs. Some tumor locations, such as the orbit, manifest early in the course of the illness. Other tumors, such as those of the retroperitoneal area, only produce symptoms when they are relatively large and compress adjacent organs. Unfortunately, many of the signs and symptoms attributable to rhabdomyosarcoma are vague and frequently suggest a common childhood illness, such as earache or runny nose. An abdominal mass, sore throat and ear pain, and ecchymosis of conjunctiva are signs of a rhabdomyosarcoma tumor. Bone fractures would be seen in osteosarcoma and a headache is a sign of a brain tumor

The nurse is monitoring an infant for signs of increased intracranial pressure (ICP). Which are late signs of increased ICP in an infant (Select all that apply)? a. Tachycardia b. Alteration in pupil size and reactivity c. Increased motor response d. Extension or flexion posturing e. Cheyne-Stokes respirations

ANS: B, D, E Late signs of ICP in an infant or child include bradycardia, alteration in pupil size and reactivity, decreased motor response, extension or flexion posturing, and Cheyne-Stokes respirations.

Which neurologic diagnostic test gives a visualized horizontal and vertical cross section of the brain at any axis? a. Nuclear brain scan c. Computed tomography (CT) scan b. Echoencephalography d. Magnetic resonance imaging (MRI)

ANS: C A CT scan provides visualization of the horizontal and vertical cross sections of the brain at any axis. A nuclear brain scan uses a radioisotope that accumulates where the blood-brain barrier is defective. Echoencephalography identifies shifts in midline structures of the brain as a result of intracranial lesions. MRI permits visualization of morphologic features of target structures and tissue discrimination that is unavailable with any other techniques.

Which statement is most descriptive of a concussion? a. Petechial hemorrhages cause amnesia. b. Visible bruising and tearing of cerebral tissue occur. c. It is a transient, reversible neuronal dysfunction. d. A slight lesion develops remote from the site of trauma

ANS: C A concussion is a transient, reversible neuronal dysfunction with instantaneous loss of awareness and responsiveness resulting from trauma to the head. Petechial hemorrhages along the superficial aspects of the brain along the point of impact are a type of contusion but are not necessarily associated with amnesia. A contusion is visible bruising and tearing of cerebral tissue. Contrecoup is a lesion that develops remote from the site of trauma as a result of an acceleration/deceleration injury

A 10-year-old boy has been hit by a car while riding his bicycle in front of the school. The school nurse immediately assesses airway, breathing, and circulation. The next nursing action should be to: a. Place on side. c. Stabilize neck and spine. b. Take blood pressure. d. Check scalp and back for bleeding.

ANS: C After determining that the child is breathing and has adequate circulation, the next action is to stabilize the neck and spine to prevent any additional trauma. The childs position should not be changed until the neck and spine are stabilized. Blood pressure is a later assessment. Less urgent, but an important assessment, is inspection of the scalp for bleeding.

. A 3-year-old child is hospitalized after a near-drowning accident. The childs mother complains to the nurse, This seems unnecessary when he is perfectly fine. The nurses best reply is: a. He still needs a little extra oxygen. b. Im sure he is fine, but the doctor wants to make sure. Test Bank - Maternal Child Nursing Care by Perry (6th Edition, 2017) 780 c. The reason for this is that complications could still occur. d. It is important to observe for possible central nervous system problems

ANS: C All children who have a near-drowning experience should be admitted to the hospital for observation. Although many children do not appear to have suffered adverse effects from the event, complications such as respiratory compromise and cerebral edema may occur up to 24 hours after the incident. Stating that, He still needs a little extra oxygen does not respond directly to the mothers concern. Why is her child still receiving oxygen? The nurse should clarify that different complications can occur up to 24 hours later and that observations are necessary.

Which statement is accurate concerning a childs musculoskeletal system and how it may be different from an adults? a. Growth occurs in children as a result of an increase in the number of muscle fibers. b. Infants are at greater risk for fractures because their epiphyseal plates are not fused. c. Because soft tissues are resilient in children, dislocations and sprains are less common than in adults. d. Childrens bones have less blood flow.

ANS: C Because soft tissues are resilient in children, dislocations and sprains are less common than in adults. A childs growth occurs because of an increase in size rather than an increase in the number of the muscle fibers. Fractures in children younger than 1 year are unusual because a large amount of force is necessary to fracture their bones. A childs bones have greater blood flow than an adults bones.

Which type of seizure involves both hemispheres of the brain? a. Focal c. Generalized b. Partial d. Acquired

ANS: C Clinical observations of generalized seizures indicate that the initial involvement is from both hemispheres. Test Bank - Maternal Child Nursing Care by Perry (6th Edition, 2017) 784 Focal seizures may arise from any area of the cerebral cortex, but the frontal, temporal, and parietal lobes are most commonly affected. Partial seizures are caused by abnormal electrical discharges from epileptogenic foci limited to a circumscribed region of the cerebral cortex. A seizure disorder that is acquired is a result of a brain injury from a variety of factors; it does not specify the type of seizure

A young childs parents call the nurse after their child was bitten by a raccoon in the woods. The nurses recommendation should be based on knowing that: a. The child should be hospitalized for close observation. Test Bank - Maternal Child Nursing Care by Perry (6th Edition, 2017) 783 b. No treatment is necessary if thorough wound cleaning is done. c. Antirabies prophylaxis must be initiated. d. Antirabies prophylaxis must be initiated if clinical manifestations appear.

ANS: C Current therapy for a rabid animal bite consists of a thorough cleansing of the wound and passive immunization with human rabies immune globulin (HRIG) as soon as possible. Hospitalization is not necessary. The wound cleansing, passive immunization, and immune globulin administration can be done as an outpatient. The child needs to receive both HRIG and rabies vaccine.

When infants are seen for fractures, which nursing intervention is a priority? a. No intervention is necessary. It is not uncommon for infants to fracture bones. b. Assess the familys safety practices. Fractures in infants usually result from falls. c. Assess for child abuse. Fractures in infants are often nonaccidental. d. Assess for genetic factors.

ANS: C Fractures in infants warrant further investigation to rule out child abuse. Fractures in children younger than 1 year are unusual because of the cartilaginous quality of the skeleton; a large amount of force is necessary to fracture their bones. Infants should be cared for in a safe environment and should not be falling. Fractures in infancy are usually nonaccidental rather than related to a genetic factor.

The most common clinical manifestation of brain tumors in children is: a. Irritability. c. Headaches and vomiting. b. Seizures. d. Fever and poor fine motor control

ANS: C Headaches, especially on awakening, and vomiting that is not related to feeding are the most common clinical manifestations of brain tumors in children. Irritability, seizures, and fever and poor fine motor control are clinical manifestations of brain tumors, but headaches and vomiting are the most common

When does idiopathic scoliosis become most noticeable? a. Newborn period b. When child starts to walk c. During preadolescent growth spurt d. Adolescence

ANS: C Idiopathic scoliosis is most noticeable during the preadolescent growth spurt and is seldom apparent before age 10 years.

The nurse is teaching a family how to care for their infant in a Pavlik harness to treat developmental dysplasia of the hip. What should be included? a. Apply lotion or powder to minimize skin irritation. b. Remove the harness several times a day to prevent contractures. c. Return to the clinic every 1 to 2 weeks. d. Place a diaper over harness, preferably using a superabsorbent disposable diaper that is relatively thin.

ANS: C Infants have a rapid growth pattern. The child needs to be assessed by the practitioner every 1 to 2 weeks for possible adjustments. Lotions and powders should not be used with the harness. The harness should not be removed, except as directed by the practitioner. A thin disposable diaper can be placed under the harness

The initial clinical manifestation of generalized seizures is: a. Being confused. c. Losing consciousness. b. Feeling frightened. d. Seeing flashing lights.

ANS: C Loss of consciousness is a frequent occurrence in generalized seizures and is the initial clinical manifestation. Being confused, feeling frightened, and seeing flashing lights are clinical manifestations of a complex partial seizure.

Which term is used to describe a childs level of consciousness when the child can be aroused with stimulation? a. Stupor c. Obtundation b. Confusion d. Disorientation

ANS: C Obtundation describes a level of consciousness in which the child can be aroused with stimulation. Stupor is a state in which the child remains in a deep sleep, responsive only to vigorous and repeated stimulation. Confusion is impaired decision making. Disorientation is confusion regarding time and plac

An appropriate nursing intervention when caring for an unconscious child should be to: a. Change the childs position infrequently to minimize the chance of increased intracranial pressure (ICP). b. Avoid using narcotics or sedatives to provide comfort and pain relief. c. Monitor fluid intake and output carefully to avoid fluid overload and cerebral edema. d. Give tepid sponge baths to reduce fever because antipyretics are contraindicated

ANS: C Often comatose patients cannot cope with the quantity of fluids that they normally tolerate. Overhydration must be avoided to prevent fatal cerebral edema. The childs position should be changed frequently to avoid complications such as pneumonia and skin breakdown. Narcotics and sedatives should be used as necessary to reduce pain and discomfort, which can increase ICP. Antipyretics are the method of choice for fever reduction

The primary method of treating osteomyelitis is: a. Joint replacement. b. Bracing and casting. c. Intravenous antibiotic therapy. d. Long-term corticosteroid therapy

ANS: C Osteomyelitis is an infection of the bone, most commonly caused by Staphylococcus aureus. The treatment of choice is antibiotics. Joint replacement, bracing and casting, and long-term corticosteroids are not indicated for infectious processes.

Which type of traction uses skin traction on the lower leg and a padded sling under the knee? a. Dunlop b. Bryants c. Russell d. Bucks extension

ANS: C Russell traction uses skin traction on the lower leg and a padded sling under the knee. The combination of longitudinal and perpendicular traction allows realignment of the lower extremity and immobilizes the hips and knees in a flexed position. Dunlop traction is an upper extremity traction used for fractures of the humerus. Bryants traction is skin traction with the legs flexed at a 90-degree angle at the hip. Bucks extension traction is Test Bank - Maternal Child Nursing Care by Perry (6th Edition, 2017) 814 a type of skin traction with the legs in an extended position. It is used primarily for short-term immobilization, before surgery with dislocated hips, for correcting contractures, or for bone deformities such as Legg-Calv- Perthes disease

A child is brought to the emergency department after experiencing a seizure at school. There is no previous history of seizures. The father tells the nurse that he cannot believe the child has epilepsy. The nurses best response is: a. Epilepsy is easily treated. b. Very few children have actual epilepsy. c. The seizure may or may not mean that your child has epilepsy. d. Your child has had only one convulsion; it probably wont happen again

ANS: C Seizures are the indispensable characteristic of epilepsy; however, not every seizure is epileptic. Epilepsy is a chronic seizure disorder with recurrent and unprovoked seizures. The treatment of epilepsy involves a thorough assessment to determine the type of seizure the child is having and the cause of events, followed by individualized therapy to allow the child to have as normal a life as possible. The nurse should not make generalized comments like Very few children have actual epilepsy and Your child has had only one convulsion; it probably wont happen again until further assessment is made

. A neonate is born with mild clubfeet. When the parents ask the nurse how this will be corrected, the nurse should explain that: a. Traction is tried first. b. Surgical intervention is needed. c. Frequent, serial casting is tried first. d. Children outgrow this condition when they learn to walk.

ANS: C Serial casting, the preferred treatment, is begun shortly after birth before discharge from the nursery. Successive casts allows for gradual stretching of skin and tight structures on the medial side of the foot. Manipulation and casting of the leg are repeated frequently (every week) to accommodate the rapid growth of early infancy. Surgical intervention is done only if serial casting is not successful. Children do not improve without intervention.

Which type of fracture describes traumatic separation of cranial sutures? a. Basilar c. Diastatic b. Compound d. Depressed

ANS: C Test Bank - Maternal Child Nursing Care by Perry (6th Edition, 2017) 777 Diastatic skull fractures are traumatic separations of the cranial sutures. A basilar fracture involves the basilar portion of the frontal, ethmoid, sphenoid, temporal, or occipital bone. A compound fracture has the bone exposed through the skin. A depressed fracture has the bone pushed inward, causing pressure on the brain

A child has been seizure-free for 2 years. A father asks the nurse how much longer the child will need to take the antiseizure medications. The nurse includes which intervention in the response? a. Medications can be discontinued at this time. b. The child will need to take the drugs for 5 years after the last seizure. c. A stepwise approach will be used to reduce the dosage gradually. d. Seizure disorders are a lifelong problem. Medications cannot be discontinued.

ANS: C Test Bank - Maternal Child Nursing Care by Perry (6th Edition, 2017) 789 A predesigned protocol is used to wean a child gradually off antiseizure medications, usually when the child is seizure-free for 2 years and has a normal electroencephalogram. Medications must be gradually reduced to minimize the recurrence of seizures. Seizure medications can be safely discontinued. The risk of recurrence is greatest within the first year.

A home care nurse is caring for an adolescent with a T1 spinal cord injury. The adolescent suddenly becomes flushed, hypertensive, and diaphoretic. Which intervention should the nurse perform first? a. Place the adolescent in a flat right side-lying position. b. Place a cool washcloth on the adolescents forehead and continue to monitor the blood pressure. c. Implement a standing prescription to empty the bladder with a sterile in-and-out Foley catheter. d. Take a full set of vital signs and notify the health care provider.

ANS: C The adolescent is experiencing an autonomic dysreflexia episode. The paralytic nature of autonomic function is replaced by autonomic dysreflexia, especially when the lesions are above the mid-thoracic level. This autonomic phenomenon is caused by visceral distention or irritation, particularly of the bowel or bladder. Sensory impulses are triggered and travel to the cord lesion, where they are blocked, which causes activation of sympathetic reflex action with disturbed central inhibitory control. Excessive sympathetic activity is manifested by a flushing face, sweating forehead, pupillary constriction, marked hypertension, headache, and bradycardia. The precipitating stimulus may be merely a full bladder or rectum or other internal or external sensory input. It can be a catastrophic event unless the irritation is relieved. Positioning the adolescent, placing a cool washcloth on the adolescents forehead, continuing to monitor blood pressure and vital signs, and notifying the health care provider would not reverse the sympathetic reflex situation

An important nursing consideration when caring for a child with juvenile idiopathic arthritis (JIA) is to: a. Apply ice packs to relieve stiffness and pain. b. Administer acetaminophen to reduce inflammation. c. Teach child and family the correct administration of medications. d. Encourage range-of-motion exercises during periods of inflammation

ANS: C The management of JIA is primarily pharmacologic. The family should be instructed regarding administration of medications and the value of a regular schedule of administration to maintain a satisfactory blood level in the body. They need to know that nonsteroidal antiinflammatory drugs should not be given on an empty stomach and to be alert for signs of toxicity. Warm, moist heat is best for relieving stiffness and pain. Acetaminophen does not have antiinflammatory effects. Range-of-motion exercises should not be done during periods of inflammation.

A 5-year-old girl sustained a concussion when she fell out of a tree. In preparation for discharge, the nurse is discussing home care with her mother. Which statement made by the mother indicates a correct understanding of the teaching? a. I should expect my child to have a few episodes of vomiting. b. If I notice sleep disturbances, I should contact the physician immediately. c. I should expect my child to have some behavioral changes after the accident. d. If I notice diplopia, I will have my child rest for 1 hour

ANS: C The parents are advised of probably post-traumatic symptoms that may be expected, including behavioral changes. If the child has episodes of vomiting, sleep disturbances, or diplopia, they should be immediately reported for evaluation

A nurse is conducting discharge teaching for parents of an infant with osteogenesis imperfecta (OI). Further teaching is indicated if the parents make which statement? a. We will be very careful handling the baby. b. We will lift the baby by the buttocks when diapering. c. Were glad there is a cure for this disorder. d. We will schedule follow-up appointments as instructed.

ANS: C The treatment for OI is primarily supportive. Although patients and families are optimistic about new research advances, there is no cure. The use of bisphosphonate therapy with IV pamidronate to promote increased bone density and prevent fractures has become standard therapy for many children with OI; however, long bones are weakened by prolonged treatment. Infants and children with this disorder require careful handling to prevent fractures. They must be supported when they are being turned, positioned, moved, and held. Even changing a diaper may cause a fracture in severely affected infants. These children should never be held by the ankles when being diapered but should be gently lifted by the buttocks or supported with pillows. Follow-up appointments for treatment with bisphosphonate can be expected.

A current recommendation to prevent neural tube defects is the supplementation of: a. Vitamin A throughout pregnancy. b. Multivitamin preparations as soon as pregnancy is suspected. c. Folic acid for all women of childbearing age. d. Folic acid during the first and second trimesters of pregnancy

ANS: C The widespread use of folic acid among women of childbearing age is expected to decrease the incidence of spina bifida significantly. Vitamin A and multivitamin preparations do not have a relation to the prevention of spina bifida. Folic acid supplementation is recommended for the preconceptual period and during the pregnancy. Only 42% of women actually follow these guidelines.

The nurse is admitting a child with Werdnig-Hoffmann disease (spinal muscular atrophy type 1). Which signs and symptoms are associated with this disease? a. Spinal muscular atrophy b. Neural atrophy of muscles c. Progressive weakness and wasting of skeletal muscle d. Pseudohypertrophy of certain muscle groups

ANS: C Werdnig-Hoffmann disease (spinal muscular atrophy type 1) is the most common paralytic form of floppy infant syndrome (congenital hypotonia). It is characterized by progressive weakness and wasting of skeletal muscle caused by degeneration of anterior horn cells. Kugelberg-Welander syndrome is a juvenile spinal muscular atrophy with a later onset. Charcot-Marie-Tooth disease is a form of progressive neural atrophy of muscles supplied by the peroneal nerves. Progressive weakness of the distal muscles of the arms and feet is found. Duchennes muscular dystrophy is characterized by muscles, especially in the calves, thighs, and upper arms, that become enlarged from fatty infiltration and feel unusually firm or woody on palpation. The term pseudohypertrophy is derived from this muscular enlargement.

The nurse is caring for a neonate with suspected meningitis. Which clinical manifestations should the nurse prepare to assess if meningitis is confirmed (Select all that apply)? a. Headache b. Photophobia c. Bulging anterior fontanel d. Weak cry e. Poor muscle tone

ANS: C, D, E Assessment findings in a neonate with meningitis include bulging anterior fontanel, weak cry, and poor muscle tone. Headache and photophobia are signs seen in an older child.

Clinical manifestations of increased intracranial pressure (ICP) in infants are (Select all that apply): a. Low-pitched cry. b. Sunken fontanel. c. Diplopia and blurred vision. d. Irritability. e. Distended scalp veins. f. Increased blood pressure

ANS: C, D, E Diplopia and blurred vision, irritability, and distended scalp veins are signs of increased ICP in infants. Low- pitched cry, sunken fontanel, and increased blood pressure are not clinical manifestations associated with ICP in infants.

The nurse is caring for a preschool child with a cast applied recently for a fractured tibia. Which assessment findings indicate possible compartment syndrome (Select all that apply)? a. Palpable distal pulse b. Capillary refill to extremity of <3 seconds c. Severe pain not relieved by analgesics d. Tingling of extremity e. Inability to move extremity

ANS: C, D, E Indications of compartment syndrome are severe pain not relieved by analgesics, tingling of extremity, and inability to move extremity. A palpable distal pulse and capillary refill to the extremity of <3 seconds are expected findings.

The nurse is preparing to admit a newborn with myelomeningocele to the neonatal intensive care nursery. Which describes this newborns defect? a. Fissure in the spinal column that leaves the meninges and the spinal cord exposed b. Herniation of the brain and meninges through a defect in the skull c. Hernial protrusion of a saclike cyst of meninges with spinal fluid but no neural elements d. Visible defect with an external saclike protrusion containing meninges, spinal fluid, and nerves

ANS: D A myelomeningocele is a visible defect with an external saclike protrusion, containing meninges, spinal fluid, and nerves. Rachischisis is a fissure in the spinal column that leaves the meninges and the spinal cord exposed. Encephalocele is a herniation of brain and meninges through a defect in the skull, producing a fluid-filled sac. Meningocele is a hernial protrusion of a saclike cyst of meninges with spinal fluid, but no neural elements

Which nursing intervention is appropriate to assess for neurovascular competency in a child who fell off the monkey bars at school and hurt his arm? a. The degree of motion and ability to position the extremity. b. The length, diameter, and shape of the extremity. c. The amount of swelling noted in the extremity and pain intensity. d. The skin color, temperature, movement, sensation, and capillary refill of the extremity.

ANS: D A neurovascular evaluation includes assessing skin color and temperature, ability to move the affected Test Bank - Maternal Child Nursing Care by Perry (6th Edition, 2017) 820 extremity, degree of sensation experienced, and speed of capillary refill in the extremity. The degree of motion in the affected extremity and ability to position the extremity are incomplete assessments of neurovascular competency. The length, diameter, and shape of the extremity are not assessment criteria in a neurovascular evaluation. Although the amount of swelling is an important factor in assessing an extremity, it is not a criterion for a neurovascular assessment.

What action may be beneficial in reducing the risk of Reyes syndrome? a. Immunization against the disease b. Medical attention for all head injuries c. Prompt treatment of bacterial meningitis Test Bank - Maternal Child Nursing Care by Perry (6th Edition, 2017) 782 d. Avoidance of aspirin and ibuprofen for children with varicella or those suspected of having influenza

ANS: D Although the etiology of Reyes syndrome is obscure, most cases follow a common viral illness, either varicella or influenza. A potential association exists between aspirin therapy and the development of Reyes syndrome; thus use of aspirin is avoided. No immunization currently exists for Reyes syndrome. Reyes syndrome is not correlated with head injuries or bacterial meningitis.

Therapeutic management of a child with tetanus includes the administration of: a. Nonsteroidal antiinflammatory drugs (NSAIDs) to reduce inflammation. b. Muscle stimulants to counteract muscle weakness. c. Bronchodilators to prevent respiratory complications. d. Antibiotics to control bacterial proliferation at the site of injury.

ANS: D Antibiotics are administered to control the proliferation of the vegetative forms of the organism at the site of infection. Tetanus toxin acts at the myoneural junction to produce muscular stiffness and lowers the threshold for reflex excitability. NSAIDs are not routinely used. Sedatives or muscle relaxants are used to help reduce titanic spasm and prevent seizures. Respiratory status is carefully evaluated for any signs of distress because muscle relaxants, opioids, and sedatives that may be prescribed may cause respiratory depression. Bronchodilators would not be used unless specifically indicated

The parents of a child with cerebral palsy ask the nurse if any drugs can decrease their childs spasticity. The nurses response should be based on knowing that: a. Anticonvulsant medications are sometimes useful for controlling spasticity. b. Medications that would be useful in reducing spasticity are too toxic for use with children. c. Many different medications can be highly effective in controlling spasticity. d. Implantation of a pump to deliver medication into the intrathecal space to decrease spasticity has recently become available

ANS: D Baclofen given intrathecally is best suited for children with severe spasticity that interferes with activities of daily living and ambulation. Anticonvulsant medications are used when seizures occur in children with cerebral palsy. The intrathecal route decreases the side effects of the drugs that reduce spasticity. Few medications are presently available for the control of spasticity.

The mother of a 1-month-old infant tells the nurse that she worries that her baby will get meningitis like her oldest son did when he was an infant. The nurse should base her response on knowing that: a. Meningitis rarely occurs during infancy. b. Often a genetic predisposition to meningitis is found. c. Vaccination to prevent all types of meningitis is now available. d. Vaccination to prevent Haemophilus influenzae type b meningitis has decreased the frequency of this disease in children.

ANS: D H. influenzae type B meningitis has virtually been eradicated in areas of the world where the vaccine is administered routinely. Bacterial meningitis remains a serious illness in children. It is significant because of the residual damage caused by undiagnosed and untreated or inadequately treated cases. The leading causes of neonatal meningitis are the group B streptococci and Escherichia coli organisms. Meningitis is an extension of a variety of bacterial infections. No genetic predisposition exists. Vaccinations are not available for all of the potential causative organisms

. What would cause a nurse to suspect that an infection has developed under a cast? a. Complaint of paresthesia b. Cold toes c. Increased respirations d. Hot spots felt on cast surface

ANS: D If hot spots are felt on the cast surface, they usually indicate infection beneath the area. This should be reported so a window can be made in the cast to observe the site. The five Ps of ischemia from a vascular injury include pain, pallor, pulselessness, paresthesia, and paralysis. Paresthesia is an indication of vascular injury, not infection. Cold toes may be indicative of too tight a cast and need further evaluation. Increased respirations may indicate a respiratory infection or pulmonary emboli. This should be reported, and the child should be evaluated.

Which term is used to describe an abnormally increased convex angulation in the curvature of the thoracic spine? a. Scoliosis b. Ankylosis c. Lordosis d. Kyphosis

ANS: D Kyphosis is an abnormally increased convex angulation in the curve of the thoracic spine. Scoliosis is a complex spinal deformity usually involving lateral curvature, spinal rotation causing rib asymmetry, and thoracic hypokyphosis. Ankylosis is the immobility of a joint. Lordosis is an accentuation of the cervical or lumbar curvature beyond physiologic limits.

The nurse should recommend medical attention if a child with a slight head injury experiences: a. Sleepiness. c. Headache, even if slight. b. Vomiting, even once. d. Confusion or abnormal behavior.

ANS: D Medical attention should be sought if the child exhibits confusion or abnormal behavior; loses consciousness; or has amnesia, fluid leaking from the nose or ears, blurred vision, or unsteady gait. Sleepiness alone does not require evaluation. If the child is difficult to arouse from sleep, medical attention should be obtained. Vomiting more than three times requires medical attention. Severe or worsening headache or one that interferes with sleep should be evaluated.

Which medication is usually tried first when a child is diagnosed with juvenile idiopathic arthritis (JIA)? a. Aspirin b. Corticosteroids c. Cytotoxic drugs such as methotrexate d. Nonsteroidal antiinflammatory drugs (NSAIDs)

ANS: D NSAIDs are the first drugs used in JIA. Naproxen, ibuprofen, and tolmetin are approved for use in children. Aspirin, once the drug of choice, has been replaced by the NSAIDs because they have fewer side effects and easier administration schedules. Corticosteroids are used for life-threatening complications, incapacitating arthritis, and uveitis. Methotrexate is a second-line therapy for JIA.

The nurse is caring for a child with severe head trauma after a car accident. Which is an ominous sign that often precedes death? a. Papilledema c. Dolls head maneuver b. Delirium d. Periodic and irregular breathing

ANS: D Periodic or irregular breathing is an ominous sign of brainstem (especially medullary) dysfunction that often precedes complete apnea. Papilledema is edema and inflammation of the optic nerve. It is commonly a sign of increased intracranial pressure. Delirium is a state of mental confusion and excitement marked by disorientation to time and place. The dolls head maneuver is a test for brainstem or oculomotor nerve dysfunction

The nurse is performing a Glasgow Coma Scale (GCS) on a school-age child with a head injury. The child opens eyes spontaneously, obeys commands, and is oriented to person, time, and place. Which is the score the nurse should record? a. 8 c. 13 Test Bank - Maternal Child Nursing Care by Perry (6th Edition, 2017) 787 b. 11 d. 15

ANS: D The GCS consists of a three-part assessment: eye opening, verbal response, and motor response. Numeric values of 1 through 5 are assigned to the levels of response in each category. The sum of these numeric values provides an objective measure of the patients level of consciousness (LOC). A person with an unaltered LOC would score the highest, 15. The child who opens eyes spontaneously, obeys commands, and is oriented is scored at a 15.

How should the nurse explain positioning for a lumbar puncture to a 5-year-old child? a. You will be on your knees with your head down on the table. b. You will be able to sit up with your chin against your chest. c. You will be on your side with the head of your bed slightly raised. d. You will lie on your side and bend your knees so that they touch your chin.

ANS: D The child should lie on his or her side with knees bent and chin tucked in to the knees. This position exposes the area of the back for the lumbar puncture. The knee-chest position is not appropriate for a lumbar puncture. An infant can be placed in a sitting position with the infant facing the nurse and the head steadied against the nurses body. A side-lying position with the head of the bed elevated is not appropriate for a lumbar puncture

What is the most appropriate nursing response to the father of a newborn infant with myelomeningocele who asks about the cause of this condition? a. One of the parents carries a defective gene that causes myelomeningocele. b. A deficiency in folic acid in the father is the most likely cause. c. Offspring of parents who have a spinal abnormality are at greater risk for myelomeningocele. d. There may be no definitive cause identified

ANS: D The etiology of most neural tube defects is unknown in most cases. There may be a genetic predisposition or a viral origin, and the disorder has been linked to maternal folic acid deficiency; however, the actual cause has not been determined. There is no evidence that children who have parents with spinal problems are at greater risk for neural tube defects.

A 4-year-old child is newly diagnosed with Legg-Calv-Perthes disease. Nursing considerations should include which action? a. Encouraging normal activity for as long as is possible b. Explaining the cause of the disease to the child and family c. Preparing the child and family for long-term, permanent disabilities d. Teaching the family the care and management of the corrective appliance

ANS: D The family needs to learn the purpose, function, application, and care of the corrective device and the importance of compliance to achieve the desired outcome. The initial therapy is rest and nonweight bearing, which helps reduce inflammation and restore motion. Legg-Calv-Perthes is a disease with an unknown etiology. A disturbance of circulation to the femoral capital epiphysis produces an ischemic aseptic necrosis of the femoral head. The disease is self-limiting, but the ultimate outcome of therapy depends on early and efficient therapy and the childs age at onset.

The nurse is teaching the parents of a 7-year-old child who has just had a cast applied for a fractured arm with the wrist and elbow immobilized. Which instructions should be included in the teaching? a. Swelling of the fingers is to be expected for the next 48 hours. b. Immobilize the shoulder to decrease pain in the arm. c. Allow the affected limb to hang down for 1 hour each day. d. Elevate casted arm when resting and when sitting up

ANS: D The injured extremity should be kept elevated while resting and in a sling when upright. This will increase venous return. Swelling of the fingers may indicate neurovascular damage and should be reported immediately. Permanent damage can occur within 6 to 8 hours. Joints above and below the cast on the affected extremity should be moved. The child should not engage in strenuous activity for the first few days. Rest with elevation of the extremity is encouraged

The nurse is assessing a child who was just admitted to the hospital for observation after a head injury. The most essential part of the nursing assessment to detect early signs of a worsening condition is: a. Posturing. c. Focal neurologic signs. b. Vital signs. d. Level of consciousness.

ANS: D The most important nursing observation is assessment of the childs level of consciousness. Alterations in consciousness appear earlier in the progression of head injury than do alterations of vital signs or focal neurologic signs. Neurologic posturing indicates neurologic damage. Vital signs and focal neurologic signs are later signs of progression when compared with level-of-consciousness changes.

The nurse is caring for an intubated infant with botulism in the pediatric intensive care unit. Which health care provider prescriptions should the nurse clarify with the health care provider before implementing? a. Administer 250 mg botulism immune globulin intravenously (BIG-IV) one time. b. Provide total parenteral nutrition (TPN) at 25 mL/hr intravenously. c. Titrate oxygen to keep pulse oximetry saturations greater than 92. d. Administer gentamicin sulfate (Garamycin) 10 mg per intravenous piggyback every 12 hours.

ANS: D The nurse should clarify the administration of an aminoglycoside antibiotic. Antibiotic therapy is not part of the management of infant botulism because the botulinum toxin is an intracellular molecule, and antibiotics would not be effective; aminoglycosides in particular should not be administered because they may potentiate the blocking effects of the neurotoxin. Treatment consists of immediate administration of botulism immune globulin intravenously (BIG-IV) without delaying for laboratory diagnosis. Early administration of BIG-IV neutralizes the toxin and stops the progression of the disease. The human-derived botulism antitoxin (BIG-IV) has been evaluated and is now available nationwide for use only in infant botulism. Approximately 50% of affected infants require intubation and mechanical ventilation; therefore, respiratory support is crucial, as is nutritional support because these infants are unable to feed.

The nurse is talking to a parent with a child who has a latex allergy. Which statement by the parent would indicate a correct understanding of the teaching? a. My child will have an allergic reaction if he comes in contact with yeast products. b. My child may have an upset stomach if he eats a food made with wheat or barley. c. My child will probably develop an allergy to peanuts. d. My child should not eat bananas or kiwis.

ANS: D There are cross-reactions between latex allergies and a number of foods such as bananas, avocados, kiwi, and chestnuts. Although yeast products, wheat and barley, and peanuts are potential allergens, they are currently not known to cross-react with latex

A child is unconscious after a motor vehicle accident. The watery discharge from the nose tests positive for glucose. The nurse should recognize that this suggests: a. Diabetic coma. c. Upper respiratory tract infection. b. Brainstem injury. d. Leaking of cerebrospinal fluid (CSF).

ANS: D Watery discharge from the nose that is positive for glucose suggests leaking of CSF from a skull fracture and is not associated with diabetes or respiratory tract infection. The fluid is probably CSF from a skull fracture and does not signify whether the brainstem is involved.

. The nurse uses the palms of the hands when handling a wet cast to: a. Assess dryness of the cast. b. Facilitate easy turning. c. Keep the patients limb balanced. d. Avoid indenting the cast.

ANS: D Wet casts should be handled by the palms of the hands, not the fingers, to prevent creating pressure points. Assessing dryness, facilitating easy turning, or keeping the patients limb balanced are not reasons for using the palms of the hand rather than the fingers when handling a wet cast.

Which clinical manifestation would most suggest acute appendicitis?

Abdominal pain that is most intense at McBurney's point

The nurse is preparing to care for an infant returning from pyloromyotomy surgery. Which prescribed orders should the nurse anticipate implementing (Select all that apply)?

Administration of analgesics for pain Intravenous (IV) fluids continued until tolerating fluids by mouth Clear liquids as the first feeding

Acute diarrhea is often caused by:

Antibiotic therapy.

Approach behaviors are coping mechanisms that result in a family's movement toward adjustment and resolution of the crisis of having a child with a chronic illness or disability. What is considered an approach behavior in parents?

Anticipate future problems and seek guidance and answers

Constipation has recently become a problem for a school-age girl. She is healthy except for seasonal allergies, which are now being successfully treated with antihistamines. The nurse should suspect that the constipation is most likely caused by

Antihistamines.

A child is admitted with acute glomerulonephritis. The nurse would expect the urinalysis during this acute phase to show: a. Bacteriuria and hematuria. b. Hematuria and proteinuria. c. Bacteriuria and increased specific gravity. Test Bank - Maternal Child Nursing Care by Perry (6th Edition, 2017) 761 d. Proteinuria and decreased specific gravity.

B

A school-age child with chronic renal failure is admitted to the hospital with a serum potassium level of 5.2 mEq/L. Which prescribed medication should the nurse plan to administer? a. Spironolactone (Aldactone) b. Sodium polystyrene sulfonate (Kayexalate) c. Lactulose (Cephulac) d. Calcium carbonate (Calcitab)

B

An objective of care for the child with nephrosis is to: a. Reduce blood pressure. b. Reduce excretion of urinary protein. c. Increase excretion of urinary protein. d. Increase ability of tissues to retain fluid

B

Calcium carbonate is given with meals to a child with chronic renal disease. The purpose of this is to: a. Prevent vomiting. b. Bind phosphorus. c. Stimulate appetite. d. Increase absorption of fat-soluble vitamins

B

One of the clinical manifestations of chronic renal failure is uremic frost. What best describes this term? a. Deposits of urea crystals in urine b. Deposits of urea crystals on skin c. Overexcretion of blood urea nitrogen d. Inability of body to tolerate cold temperatures

B

The diet of a child with nephrosis usually includes: a. High protein. c. Low fat. b. Salt restriction. d. High carbohydrate

B

The narrowing of preputial opening of foreskin is called: a. Chordee c. Epispadias b. Phimosis d. Hypospadias

B

The narrowing of the preputial opening of the foreskin is called: a. Chordee. c. Epispadias. b. Phimosis. d. Hypospadias

B

The nurse is conducting teaching for an adolescent being discharged to home after a renal transplantation. The adolescent needs further teaching if which statement is made? a. I will report any fever to my primary health care provider. b. I am glad I only have to take the immunosuppressant medication for two weeks. Test Bank - Maternal Child Nursing Care by Perry (6th Edition, 2017) 769 c. I will observe my incision for any redness or swelling. d. I wont miss doing kidney dialysis every week

B

Which factor predisposes a child to urinary tract infections? a. Increased fluid intake b. Short urethra in young girls c. Prostatic secretions in males d. Frequent emptying of the bladder

B

Which statement is descriptive of renal transplantation in children? a. It is an acceptable means of treatment after age 10 years. b. It is preferred means of renal replacement therapy in children. c. Children can receive kidneys only from other children. d. The decision for transplantation is difficult since a relatively normal lifestyle is not possible

B

A school-age child has been admitted to the hospital with an exacerbation of nephrotic syndrome. Which clinical manifestations should the nurse expect to assess (Select all that apply)? a. Weight loss b. Facial edema c. Cloudy, smoky browncolored urine d. Fatigue e. Frothy-appearing urine

B D E

Which interventions should the nurse plan when caring for a child with a visual impairment (select all that apply)? a. Touch the child upon entering the room before speaking. b. Keep items in the room in the same location. c. Describe the placement of the eating utensils on the meal tray. d. Use color examples to describe something to a child who has been blind since birth. e. Identify noises for the child.

B, C, E

A 6-year-old child is having a generalized seizure in the classroom at school. Place in order the interventions the school nurse should implement, starting with the highest-priority intervention and sequencing to the lowest-priority intervention. a. Take vital signs. b. Ease child to the floor. c. Allow child to rest. d. Turn child to the side. e. Integrate child back into the school environment.

B, D, A, C, E

A stool specimen from a child with diarrhea shows the presence of neutrophils and red blood cells. This is most suggestive of which condition?

Bacterial gastroenteritis

What should the nurse identify as major fears in the preschool child who is hospitalized with a chronic illness (select all that apply)?

Bodily injury Mutilation Being left alone

The nurse is explaining to a parent how to care for a child with vomiting associated with a viral illness. The nurse should include:

Brushing teeth or rinsing mouth after vomiting.

A major complication in a child with chronic renal failure is: a. Hypokalemia. b. Metabolic alkalosis. c. Water and sodium retention. d. Excessive excretion of blood urea nitrogen

C

The diet of a child with chronic renal failure is usually characterized as: a. High in protein. b. Low in vitamin D. c. Low in phosphorus. Test Bank - Maternal Child Nursing Care by Perry (6th Edition, 2017) 765 d. Supplemented with vitamins A, E, and K.

C

The most appropriate nursing diagnosis for the child with acute glomerulonephritis is: a. Risk for Injury related to malignant process and treatment. b. Deficient Fluid Volume related to excessive losses. c. Excess Fluid Volume related to decreased plasma filtration. d. Excess Fluid Volume related to fluid accumulation in tissues and third spaces

C

The nurse is admitting a school-age child in acute renal failure with reduced glomerular filtration rate. Which urine test is the most useful clinical indication of glomerular filtration rate? a. pH c. Creatinine clearance b. Osmolality d. Protein level

C

The nurse is teaching parents of a child with chronic renal failure (CRF) about the use of recombinant human erythropoietin (rHuEPO) subcutaneous injections. Which statement indicates the parents have understood the teaching? a. These injections will help with the hypertension. b. Were glad the injections only need to be given once a month. c. The red blood cell count should begin to improve with these injections. d. Urine output should begin to improve with these injections

C

The nurse is teaching the parent about the diet of a child experiencing severe edema associated with acute glomerulonephritis. Which information should the nurse include in the teaching? a. You will need to decrease the number of calories in your childs diet. b. Your childs diet will need an increased amount of protein. c. You will need to avoid adding salt to your childs food. d. Your childs diet will consist of low-fat, low-carbohydrate foods

C

Second priority

Call 911 and wait for the emergency response personnel to arrive.

Third priority

Call Jason's family practitioner to obtain further orders for medication.

Fourth priority

Call Jason's parents and notify them of the situation.

Which represents a common best practice in the provision of services to children with chronic or complex conditions?

Children with complex conditions are integrated into regular classrooms.

Chris, age 9 years, has several physical disabilities. His father explains to the nurse that his son concentrates on what he can rather than cannot do and is as independent as possible. The nurse's best interpretation of this is:

Chris is using an adaptive coping style.

The nurse is caring for an infant whose cleft lip was repaired. Important aspects of this infant's postoperative care include:

Cleansing of suture line, supine and side-lying positions, and arm restraints

What should the nurse stress in a teaching plan for the mother of an 11-year-old boy with ulcerative colitis?

Coping with stress and avoiding triggers

What is used to treat moderate-to-severe inflammatory bowel disease?

Corticosteroids

What is characterized by a chronic inflammatory process that may involve any part of the gastrointestinal (GI) tract from mouth to anus?

Crohn's disease

. The nurse is assisting the pediatric provider with a newborn examination. The provider notes that the infant has hypospadias. The nurse understands that hypospadias refers to: a. Absence of a urethral opening. b. Penis shorter than usual for age. c. Urethral opening along dorsal surface of penis. d. Urethral opening along ventral surface of penis.

D

A common side effect of corticosteroid therapy is: a. Fever. c. Weight loss. b. Hypertension. d. Increased appetite

D

A mother asks the nurse what would be the first indication that acute glomerulonephritis is improving. The nurses best response should be that the: a. Blood pressure will stabilize. c. Urine will be free of protein. b. Child will have more energy. d. Urinary output will increase

D

A preschool child is being admitted to the hospital with dehydration and a urinary tract infection (UTI). Which urinalysis result should the nurse expect with these conditions? a. WBC <1; specific gravity 1.008 c. WBC >2; specific gravity 1.016 b. WBC <2; specific gravity 1.025 d. WBC >2; specific gravity 1.030

D

An advantage of peritoneal dialysis is that: a. Treatments are done in hospitals. b. Protein loss is less extensive. c. Dietary limitations are not necessary. d. Parents and older children can perform treatments.

D

The most common cause of acute renal failure in children is: a. Pyelonephritis. c. Urinary tract obstruction. b. Tubular destruction. d. Severe dehydration

D

The nurse is caring for a child with acute renal failure. What clinical manifestation should he or she recognize as a sign of hyperkalemia? a. Dyspnea c. Oliguria b. Seizure d. Cardiac arrhythmia

D

The nurse is caring for an adolescent who has just started dialysis. The child seems always angry, hostile, or depressed. The nurse should recognize that this is most likely related to: a. Neurologic manifestations that occur with dialysis. b. Physiologic manifestations of renal disease. c. Adolescents having few coping mechanisms. d. Adolescents often resenting the control and enforced dependence imposed by dialysis

D

Which clinical manifestation would be seen in a child with chronic renal failure? a. Hypotension c. Hypokalemia b. Massive hematuria d. Unpleasant uremic breath odor

D

Which diagnostic finding is present when a child has primary nephrotic syndrome? a. Hyperalbuminemia b. Positive ASO titer c. Leukocytosis d. Proteinuria

D

Which intervention is appropriate when examining a male infant for cryptorchidism? a. Cooling the examiners hands c. Eliciting the cremasteric reflex b. Taking a rectal temperature d. Warming the room

D

A child with secondary enuresis who complains of dysuria or urgency should be evaluated for what condition (Select all that apply)? a. Hypocalciuria b. Nephrotic syndrome c. Glomerulonephritis d. Urinary tract infection (UTI) e. Diabetes mellitus

D E

An infant is brought to the emergency department with poor skin turgor, weight loss, lethargy, and tachycardia. This is suggestive of:

Dehydration

The nurse is caring for a neonate with a suspected tracheoesophageal fistula. Nursing care should include:

Elevating the head but giving nothing by mouth.

Which intervention will encourage a sense of autonomy in a toddler with disabilities?

Encourage independence in as many areas as possible.

A mother who intended to breastfeed has given birth to an infant with a cleft palate. Nursing interventions should include (Select all that apply):

Encouraging and helping mother to breastfeed. Recommending use of a breast pump to maintain lactation until infant can suck.

Which intervention will encourage a sense of autonomy in a toddler with disabilities?

Encouraging independence in as many areas as possible

Bismuth subsalicylate, clarithromycin, and metronidazole are prescribed for a child with a peptic ulcer to:

Eradicate Helicobacter pylori.

The parents of a child born with disabilities ask the nurse for advice about discipline. The nurse's response should be based on knowledge that discipline is:

Essential for the child.

At the time of a child's death, the nurse tells his mother, "We will miss him so much." The best interpretation of this is that the nurse is:

Expressing personal feelings of loss.

The nurse case manager is planning a care conference about a young child who has complex health care needs and will soon be discharged home. Whom should the nurse invite to the conference?

Family and key health professionals involved in child's care

Nurses must be alert for increased fluid requirements when a child has:

Fever

A parasite that causes acute diarrhea is

Giardia lamblia.

Which is the most appropriate nursing intervention to promote normalization in a school-age child with a chronic illness?

Give child as much control as possible.

Which intervention should be included in the nurse's plan of care for a 7-year-old child with encopresis who has cleared the initial impaction?

Give the child a choice of beverage to mix with a laxative.

A common parental reaction to a child with special needs is parental overprotection. Parental behavior suggestive of this includes:

Giving inconsistent discipline.

The nurse is caring for a child who has just died. The parents ask to be left alone so that they can rock their child one more time. The nurse should:

Grant their request.

Families progress through various stages of reactions when a child is diagnosed with a chronic illness or disability. After the shock phase, a period of adjustment usually follows. This is often characterized by:

Guilt and anger.

Careful hand washing before and after contact can prevent the spread of which condition in day care and school settings?

Hepatitis A

Which statements regarding hepatitis B are correct (Select all that apply)?

Hepatitis B can be prevented by hepatitis B virus vaccine. Hepatitis B can be transferred to an infant of a breastfeeding mother. The onset of hepatitis B is insidious. Immunity to hepatitis B occurs after one attack.

Which vaccine is now recommended for the immunization of all newborns?

Hepatitis B vaccine

Which type of dehydration results from water loss in excess of electrolyte loss?

Hypertonic dehydration

Which statement is most descriptive of Meckel's diverticulum?

Intestinal bleeding may be mild or profuse.

A young child is brought to the ER with severe dehydration secondary to acute diarrhea and vomiting. Therapeutic management of this child will begin with:

Intravenous fluids.

What is most descriptive of a school-age child's reaction to death?

Is very interested in funerals and burials

The earliest clinical manifestation of biliary atresia is:

Jaundice

An appropriate nursing intervention when providing comfort and support for a child whose death is imminent is to:

Limit care to essentials.

The best chance of survival for a child with cirrhosis is:

Liver transplantation.

An infant with pyloric stenosis experiences excessive vomiting that can result in:

Metabolic alkalosis.

A 3-year-old child with Hirschsprung's disease is hospitalized for surgery. A temporary colostomy will be necessary. The nurse should recognize that preparing this child psychologically is:

Necessary because it will be an adjustment.

A mother calls the clinic nurse about her 4-year-old son who has acute diarrhea. She has been giving him the antidiarrheal drug loperamide (Imodium A-D). The nurse's response should be based on knowledge that this drug is:

Not indicated.

The nurse is caring for a boy with probable intussusception. He had diarrhea before admission but, while waiting for administration of air pressure to reduce the intussusception, he passes a normal brown stool. The most appropriate nursing action is to:

Notify the practitioner.

What food choice by the parent of a 2-year-old child with celiac disease indicates a need for further teaching?

Oatmeal

Which statement best characterizes hepatitis A?

Onset is usually rapid and acute

A nurse is planning palliative care for a child with severe pain. Which should the nurse expect to be prescribed for pain relief?

Opioids on a regular schedule

Therapeutic management of the child with acute diarrhea and dehydration usually begins with:

Oral rehydration solution (ORS).

Which term best describes a multidisciplinary approach to the management of a terminal illness that focuses on symptom control and support?

Palliative care

The nurse is talking with the parent of a child newly diagnosed with a chronic illness. The parent is upset and tearful. The nurse asks, "With whom do you talk when something is worrying you?" This should be interpreted as:

Part of assessing parent's available support system.

The nurse is providing support to parents at the time their child is diagnosed with chronic disabilities. The nurse notices that the parents keep asking the same questions. The nurse should:

Patiently continue to answer questions.

Most parents of children with special needs tend to experience chronic sorrow. This is characterized by:

Periods of intensified sorrow and loss that occur in waves over time.

A high-fiber food that the nurse could recommend for a child with chronic constipation is:

Popcorn

Kelly, age 8 years, will soon be able to return to school after an injury that resulted in several severe, chronic disabilities. What is the most appropriate action by the school nurse?

Preparing Kelly's classmates and teachers for changes they can expect

The feeling of guilt that the child "caused" the disability or illness is especially critical in which child?

Preschooler

First priority

Promptly administer an intramuscular (IM) dose of epinephrine.

Which interventions should a nurse implement when caring for a child with hepatitis (Select all that apply)?

Provide a well-balanced, low-fat diet. Teach parents not to administer any over-the-counter medications. Instruct parents on the importance of good hand washing.

Caring for the newborn with a cleft lip and palate before surgical repair includes:

Providing satisfaction of sucking needs.

A preschooler is found digging up a pet bird that was recently buried after it died. The best explanation for this behavior is that

Reassurance is needed that the pet has not gone somewhere else.

A newborn was admitted to the nursery with a complete bilateral cleft lip and palate. The physician explained the plan of therapy and its expected good results. However, the mother refuses to see or hold her baby. Initial therapeutic approach to the mother should be to:

Recognize that negative feelings toward the child continue throughout childhood.

A histamine receptor antagonist such as cimetidine (Tagamet) or ranitidine (Zantac) is ordered for an infant with gastroesophageal reflux. The purpose of this is to:

Reduce gastric acid production.

An important nursing consideration in the care of a child with celiac disease is to:

Refer to a nutritionist for detailed dietary instructions and education.

Which describe avoidance behaviors a parent may exhibit when learning that his or her child has a chronic condition (select all that apply)?

Refuses to agree to treatment Withdraws from outside world Punishes self because of guilt and shame

During the first few days after surgery for cleft lip, which intervention should the nurse do?

Remove restraints periodically to cuddle infant.

What is a priority nursing diagnosis for the preschool child with chronic illness?

Risk for Delayed Growth and Development related to chronic illness or disability

The viral pathogen that frequently causes acute diarrhea in young children is:

Rotavirus

Which type of hernia has an impaired blood supply to the herniated organ?

Strangulated hernia

When caring for a child with probable appendicitis, the nurse should be alert to recognize that a sign of perforation is

Sudden relief from pain.

Nursing interventions to help the siblings of a child with a complex or chronic condition cope include:

Suggesting to the parents ways of showing gratitude to the siblings who help care for the child with a disability or chronic condition.

Therapeutic management of most children with Hirschsprung's disease is primarily:

Surgical removal of affected section of bowel.

A nurse is caring for a child who is near death. Which physical signs indicate the child is approaching death (select all that apply)?

Tactile sensation decreasing Change in respiratory pattern Difficulty swallowing

What is the most important information to be included in the discharge planning for an infant with gastroesophageal reflux?

Teach the parents how to do infant cardiopulmonary resuscitation (CPR).

What is the major focus of the therapeutic management for a child with lactose intolerance?

Teaching dietary modifications

An infant with short bowel syndrome will be discharged home on total parenteral nutrition (TPN) and gastrostomy feedings. Nursing care should include

Teaching the family signs of central venous catheter infection.

A school-age child is diagnosed with a life-threatening illness. The parents want to protect their child from knowing the seriousness of the illness. The nurse should explain that:

Terminally ill children know when they are seriously ill.

A cure is no longer possible for a young child with cancer. The nursing staff recognizes that the goal of treatment must shift from cure to palliation. Which is an important consideration at this time?

The family is included in the decision to shift the goals of treatment.

The nurse is talking with the parents of a child who died 6 months ago. They sometimes still "hear" the child's voice and have trouble sleeping. They describe feeling "empty" and depressed. The nurse should recognize that:

These are normal grief responses.

A 4-month-old infant has gastroesophageal reflux disease (GERD) but is thriving without other complications. What should the nurse suggest to minimize reflux?

Thicken formula with rice cereal.

Lindsey, age 5 years, will be starting kindergarten next month. She has cerebral palsy, and it has been determined that she needs to be in a special education classroom. Her parents are tearful when telling the nurse about this and state that they did not realize that her disability was so severe. The best interpretation of this situation is that:

This is a normal anticipated time of parental stress.

A 16-year-old boy with a chronic illness has recently become rebellious and is taking risks such as missing doses of his medication. The nurse should explain to his parents that:

This is part of normal adolescence.

The nurse is caring for an infant with suspected pyloric stenosis. Which clinical manifestation would indicate pyloric stenosis?

Visible peristalsis and weight loss

A nurse is conducting dietary teaching on high-fiber foods for parents of a child with constipation. Which foods should the nurse include as being high in fiber (Select all that apply)?

Whole grain breads Bran pancakes Raw carrots

The nurse is talking to the parent of a 13-month-old child. The mother states, "My child does not make noises like 'da' or 'na' like my sister's baby, who is only 9 months old." Which statement by the nurse would be most appropriate to make? a. "I am going to request a referral to a hearing specialist." b. "You should not compare your child to your sister's child." c. "I think your child is fine, but we will check again in 3 months." d. "You should ask other parents what noises their children made at this age."

a. "I am going to request a referral to a hearing specialist."

When should children with cognitive impairment be referred for stimulation and educational programs? a. As young as possible b. As soon as they have the ability to communicate in some way c. At age 3 years, when schools are required to provide services d. At age 5 or 6 years, when schools are required to provide services

a. As young as possible

Prevention of hearing impairment in children is a major goal for the nurse. This can be achieved through: a. Being involved in immunization clinics for children. b. Assessing a newborn for hearing loss. c. Answering parents' questions about hearing aids. d. Participating in hearing screening in the community.

a. Being involved in immunization clinics for children.

The most common type of hearing loss, which results from interference of transmission of sound to the middle ear, is called: a. Conductive. c. Mixed conductive-sensorineural. b. Sensorineural. d. Central auditory imperceptive.

a. Conductive.

A nurse would suspect possible visual impairment in a child who displays: a. Excessive rubbing of the eyes. b. Rapid lateral movement of the eyes. c. Delay in speech development. d. Lack of interest in casual conversation with peers.

a. Excessive rubbing of the eyes.

Parents have learned that their 6-year-old child has autism. The nurse may help the parents to cope by explaining that the child may: a. Have an extremely developed skill in a particular area. b. Outgrow the condition by early adulthood. c. Have average social skills. d. Have age-appropriate language skills.

a. Have an extremely developed skill in a particular area.

A child with autism spectrum disorder (ASD) is admitted to the hospital with pneumonia. The nurse should plan which priority intervention when caring for the child? a. Maintain a structured routine and keep stimulation to a minimum. b. Place the child in a room with a roommate of the same age. c. Maintain frequent touch and eye contact with the child. d. Take the child frequently to the playroom to play with other children.

a. Maintain a structured routine and keep stimulation to a minimum.

The major consideration when selecting toys for a child who is cognitively impaired is: a. Safety. b. Age appropriateness. c. Ability to provide exercise. d. Ability to teach useful skills.

a. Safety

Which action best facilitates lipreading by the hearing-impaired child? a. Speaking at an even rate b. Exaggerating pronunciation of words c. Avoiding using facial expressions d. Repeating in exactly the same way if child does not understand

a. Speaking at an even rate

At what developmental period do children have the most difficulty coping with death, particularly if it is their own?

adolescence

A nurse is providing a parent information regarding autism. Which statement made by the parent indicates understanding of the teaching? a. "Autism is characterized by periods of remission and exacerbation." b. "The onset of autism usually occurs before 3 years of age." c. "Children with autism have imitation and gesturing skills." d. "Autism can be treated effectively with medication."

b. "The onset of autism usually occurs before 3 years of age."

When a child with mild cognitive impairment reaches the end of adolescence, what characteristic would be expected? a. Achieves a mental age of 5 to 6 years b. Achieves a mental age of 8 to 12 years c. Is unable to progress in functional reading or arithmetic d. Acquires practical skills and useful reading and arithmetic to an eighth-grade level

b. Achieves a mental age of 8 to 12 years

The school nurse is caring for a child with a penetrating eye injury. Emergency treatment includes: a. Applying a regular eye patch. b. Applying a Fox shield to the affected eye and any type of patch to the other eye. c. Applying ice until the physician is seen. d. Irrigating the eye copiously with a sterile saline solution.

b. Applying a Fox shield to the affected eye and any type of patch to the other eye.

A child with autism is hospitalized with asthma. The nurse should plan care so that the: a. Parents' expectations are met. b. Child's routine habits and preferences are maintained. c. Child is supported through the autistic crisis. d. Parents need not be at the hospital.

b. Child's routine habits and preferences are maintained.

Mark, a 9-year-old with Down syndrome, is mainstreamed into a regular third-grade class for part of the school day. His mother asks the school nurse about programs such as Cub Scouts that he might join. The nurse's recommendation should be based on knowing that: a. Programs such as Cub Scouts are inappropriate for children who are cognitively impaired. b. Children with Down syndrome have the same need for socialization as other children. c. Children with Down syndrome socialize better with children who have similar disabilities. d. Parents of children with Down syndrome encourage programs such as scouting because they deny that their children have disabilities.

b. Children with Down syndrome have the same need for socialization as other children.

An implanted ear prosthesis for children with sensorineural hearing loss is a(n): a. Hearing aid. c. Auditory implant. b. Cochlear implant. d. Amplification device.

b. Cochlear implant.

A newborn assessment shows separated sagittal suture, oblique palpebral fissures, depressed nasal bridge, protruding tongue, and transverse palmar creases. These findings are most suggestive of: a. Microcephaly. c. Cerebral palsy. b. Down syndrome. d. Fragile X syndrome.

b. Down syndrome

A parent whose child has been diagnosed with a cognitive deficit should be counseled that intellectual impairment: a. Is usually due to a genetic defect. b. May be caused by a variety of factors. c. Is rarely due to first-trimester events. d. Is usually caused by parental intellectual impairment.

b. May be caused by a variety of factors.

Appropriate interventions to facilitate socialization of the cognitively impaired child include to: a. Provide age-appropriate toys and play activities. b. Provide peer experiences such as Special Olympics when older. c. Avoid exposure to strangers who may not understand cognitive development. d. Emphasize mastery of physical skills because they are delayed more often than verbal skills.

b. Provide peer experiences such as Special Olympics when older.

Distortion of sound and problems in discrimination are characteristic of which type of hearing loss? a. Conductive b. Sensorineural c. Mixed conductive-sensorineural d. Central auditory imperceptive

b. Sensorineural

A parent asks the nurse why a developmental assessment is being conducted for a child during a routine well-child visit. The nurse answers based on the knowledge that routine developmental assessments during well-child visits are: a. Not necessary unless the parents request them. b. The best method for early detection of cognitive disorders. c. Frightening to parents and children and should be avoided. d. Valuable in measuring intelligence in children.

b. The best method for early detection of cognitive disorders.

Which statement by a parent about a child's conjunctivitis indicates that further teaching is needed? a. "I'll have separate towels and washcloths for each family member." b. "I'll notify my doctor if the eye gets redder or the drainage increases." c. "When the eye drainage improves, we'll stop giving the antibiotic ointment." d. "After taking the antibiotic for 24 hours, my child can return to school."

c. "When the eye drainage improves, we'll stop giving the antibiotic ointment."

A 10-year-old patient is talking to the nurse about wanting to try contact lenses instead of wearing glasses. She states that the other children at her school call her "four-eyes." Contact lenses should be prescribed for a child who is: a. At least 12 years of age. b. Able to read all the written information and instructions. c. Able to independently care for the lenses in a responsible manner. d. Confident that she really wants contact lenses.

c. Able to independently care for the lenses in a responsible manner.

The child with Down syndrome should be evaluated for what characteristic before participating in some sports? a. Hyperflexibility b. Cutis marmorata c. Atlantoaxial instability d. Speckling of iris (Brushfield's spots)

c. Atlantoaxial instability

Which term refers to opacity of the crystalline lens that prevents light rays from entering the eye and refracting on the retina? a. Myopia b. Amblyopia c. Cataract d. Glaucoma

c. Cataract

When caring for a newborn with Down syndrome, the nurse should be aware that the most common congenital anomaly associated with Down syndrome is: a. Hypospadias. c. Congenital heart disease. b. Pyloric stenosis. d. Congenital hip dysplasia.

c. Congenital heart disease.

A nurse is preparing to perform a dressing change on a 6-year-old child with mild cognitive impairment (CI) who sustained a minor burn. Which strategy should the nurse use to prepare the child for this procedure? a. Verbally explain what will be done. b. Have the child watch a video on dressing changes. c. Demonstrate a dressing change on a doll. d. Explain the importance of keeping the burn area clean.

c. Demonstrate a dressing change on a doll.

Which action is contraindicated when a child with Down syndrome is hospitalized? a. Determine the child's vocabulary for specific body functions. b. Assess the child's hearing and visual capabilities. c. Encourage parents to leave the child alone for extended periods of time. d. Have meals served at the child's usual mealtimes.

c. Encourage parents to leave the child alone for extended periods of time.

The nurse is discussing sexuality with the parents of an adolescent girl with moderate cognitive impairment. Which should the nurse consider when dealing with this issue? a. Sterilization is recommended for any adolescent with cognitive impairment. b. Sexual drive and interest are limited in individuals with cognitive impairment. c. Individuals with cognitive impairment need a well-defined, concrete code of sexual conduct. d. Sexual intercourse rarely occurs unless the individual with cognitive impairment is sexually abused.

c. Individuals with cognitive impairment need a well-defined, concrete code of sexual conduct.

A father calls the emergency department nurse saying that his daughter's eyes burn after getting some dishwasher detergent in them. The nurse recommends that the child be seen in the emergency department or by an ophthalmologist. What should the nurse recommend before the child is transported? a. Keep the eyes closed. b. Apply cold compresses. c. Irrigate eyes copiously with tap water for 20 minutes. d. Prepare a normal saline solution (salt and water) and irrigate eyes for 20 minutes.

c. Irrigate eyes copiously with tap water for 20 minutes

A young child who has an intelligence quotient (IQ) of 45 would be described as: a. Within the lower limits of the range of normal intelligence. b. Mildly cognitively impaired but educable. c. Moderately cognitively impaired but trainable. d. Severely cognitively impaired and completely dependent on others for care.

c. Moderately cognitively impaired but trainable.

An appropriate nursing diagnosis for a child with a cognitive dysfunction who has a limited ability to anticipate danger is: a. Impaired Social Interaction. b. Deficient Knowledge. c. Risk for Injury. d. Ineffective Coping.

c. Risk for Injury.

The nurse is talking with a 10-year-old boy who wears bilateral hearing aids. The left hearing aid is making an annoying whistling sound that the child cannot hear. The most appropriate nursing action is to: a. Ignore the sound. b. Ask him to reverse the hearing aids in his ears. c. Suggest that he reinsert the hearing aid. d. Suggest that he raise the volume of the hearing aid.

c. Suggest that he reinsert the hearing aid

What should the nurse keep in mind when planning to communicate with a child who has autism? a. The child has normal verbal communication. b. The child is expected to use sign language. c. The child may exhibit monotone speech and echolalia. d. The child is not listening if she is not looking at the nurse.

c. The child may exhibit monotone speech and echolalia.

The pediatric nurse understands that fragile X syndrome is: a. A chromosome defect affecting only females. b. A chromosome defect that follows the pattern of X-linked recessive disorders. c. The second most common genetic cause of cognitive impairment. d. The most common cause of noninherited cognitive impairment.

c. The second most common genetic cause of cognitive impairment.

Which teaching guideline helps prevent eye injuries during sports and play activities? a. Restrict helmet use to those who wear eyeglasses or contact lenses. b. Discourage the use of goggles with helmets. c. Wear eye protection when participating in high-risk sports such as paintball. d. Wear a face mask when playing any sport or playing roughly.

c. Wear eye protection when participating in high-risk sports such as paintball.

The most common clinical manifestation of retinoblastoma is: a. Glaucoma. c. White eye reflex. b. Amblyopia. d. Sunken eye socket.

c. White eye reflex.

A nurse is preparing a teaching session for parents on prevention of childhood hearing loss. The nurse should include that the most common cause of hearing impairment in children is: a. Auditory nerve damage. c. Congenital rubella. b. Congenital ear defects. d. Chronic otitis media.

d. Chronic otitis media.

The teaching plan for the parents of a 3-year-old child with amblyopia ("lazy eye") should include what instruction? a. Apply a patch to the child's eyeglass lenses. b. Apply a patch only during waking hours. c. Apply a patch over the "bad" eye to strengthen it. d. Cover the "good" eye completely with a patch.

d. Cover the "good" eye completely with a patch.

A 2-year-old girl has excessive tearing and corneal haziness. The nurse knows that these symptoms may indicate: a. Viral conjunctivitis. c. Congenital cataract. b. Paralytic strabismus. c. Congenital cataract. d. Infantile glaucoma.

d. Infantile glaucoma.

An adolescent male visits his primary care provider complaining of difficulty with his vision. When the nurse asks the adolescent to explain what visual deficits he is experiencing, the adolescent states, "I am having difficulty seeing distant objects; they are less clear than things that are close." What disorder does the nurse suspect the adolescent has? a. Hyphema b. Astigmatism c. Amblyopia d. Myopia

d. Myopia

An adolescent gets hit in the eye during a fight. The school nurse, using a flashlight, notes the presence of gross hyphema (hemorrhage into anterior chamber). The nurse should: a. Apply a Fox shield. b. Instruct the adolescent to apply ice for 24 hours. c. Have adolescent rest with eye closed and ice applied. d. Notify parents that adolescent needs to see an ophthalmologist.

d. Notify parents that adolescent needs to see an ophthalmologist.

The nurse comes into the room of a child who was just diagnosed with a chronic disability. The child's parents begin to yell at the nurse about a variety of concerns. The nurse's best response is:

"Being angry is only natural."

Which description of a stool is characteristic of intussusception?

"Currant jelly" stools

What is the most appropriate response to a school-age child who asks if she can talk to her dying sister?

"Even though she will probably not answer you, she can still hear what you say to her."

The nurse and a new nurse are caring for a child who will require palliative care. Which statement made by the new nurse would indicate a correct understanding of palliative care?

"Palliative care provides pain and symptom management for the child."

Which are appropriate statements the nurse should make to parents after the death of their child (select all that apply)?

"We feel so sorry that we couldn't save your child." "You're feeling all the pain of losing a child."

A child has a nasogastric (NG) tube to continuous low intermittent suction. The physician's prescription is to replace the previous 4-hour NG output with a normal saline piggyback over a 2-hour period. The NG output for the previous 4 hours totaled 50 mL. What milliliter/hour rate should the nurse administer to replace with a normal saline piggyback? _____ Record your answer as a whole number.

25 The previous total 4-hour output was 50 mL. To run the 50 mL over a 2-hour period, the nurse would divide 50 by 2 = 25. The normal saline replacement fluid would be run at 25 mL/hr.

At what age do most children have an adult concept of death as being inevitable, universal, and irreversible?

9 to 11 years

The nurse closely monitors the temperature of a child with nephrosis. The purpose of this is to detect an early sign of: a. Infection. c. Encephalopathy. b. Hypertension. d. Edema.

A

The nurse is conducting an assessment on a school-age child with urosepsis. Which assessment finding should the nurse expect? a. Fever with a positive blood culture c. Oliguria and hypertension b. Proteinuria and edema d. Anemia and thrombocytopenia

A

The primary clinical manifestations of acute renal failure are: a. Oliguria and hypertension. c. Proteinuria and muscle cramps. b. Hematuria and pallor. d. Bacteriuria and facial edema

A

Therapeutic management of nephrosis includes: a. Corticosteroids. c. Long-term diuretics. b. Antihypertensive agents. d. Increased fluids to promote diuresis

A

What should the nurse include in a teaching plan for the parents of a child with vesicoureteral reflux? a. The importance of taking prophylactic antibiotics b. Suggestions for how to maintain fluid restrictions c. The use of bubble baths as an incentive to increase bath time d. The need for the child to hold urine for 6 to 8 hours

A

What should the nurse recommend to prevent urinary tract infections in young girls? a. Wearing cotton underpants b. Limiting bathing as much as possible c. Increasing fluids; decreasing salt intake d. Cleansing the perineum with water after voiding

A

When a child has chronic renal failure, the progressive deterioration produces a variety of clinical and biochemical disturbances that eventually are manifested in the clinical syndrome known as: a. Uremia. c. Proteinuria. b. Oliguria. d. Pyelonephritis.

A

Which diagnostic test allows visualization of the renal parenchyma and renal pelvis without exposure to external beam radiation or radioactive isotopes? a. Renal ultrasound b. Computed tomography c. Intravenous pyelography d. Voiding cystourethrography

A

The nurse is caring for an infant with a suspected urinary tract infection. Which clinical manifestations would be observed (Select all that apply)? a. Vomiting b. Jaundice c. Failure to gain weight d. Swelling of the face e. Back pain f. Persistent diaper rash

A C F

A school-age child is admitted to the hospital with acute glomerulonephritis and oliguria. Which dietary menu items should be allowed for this child (Select all that apply)? a. Apples b. Bananas c. Cheese d. Carrot sticks e. Strawberries

A D E

For what clinical manifestation should a nurse be alert when suspecting a diagnosis of esophageal atresia?

A nasogastric tube fails to pass at birth.

Which best describes how preschoolers react to the death of a loved one?

A preschooler is likely to feel guilty and responsible for the death.

Which assessment findings indicate to the nurse a child has Down syndrome (select all that apply)? a. High-arched, narrow palate b. Protruding tongue c. Long, slender fingers d. Transverse palmar crease e. Hypertonic muscle tone

A, B, D

A nurse is instructing a nursing assistant on techniques to facilitate lipreading with a hearing-impaired child who lip-reads. Which techniques should the nurse include (select all that apply)? a. Speak at eye level. b. Stand at a distance from the child. c. Speak words in a loud tone. d. Use facial expressions while speaking. e. Keep sentences short.

A, D, E

Autism is a complex developmental disorder. The diagnostic criteria for autism include delayed or abnormal functioning in which area(s) with onset before age 3 years (select all that apply)? a. Language as used in social communication b. Gross motor development c. Growth below the 5th percentile for height and weight d. Symbolic or imaginative play e. Social interaction

A, D, E

A toddler is admitted to the hospital with a possible diagnosis of tetanus. The health care provider has prescribed lorazepam (Ativan) intravenously 0.05 mg/kg/dose every 6 hours prn as a muscle relaxant. The child weighs 22 lbs. How many milligrams of Ativan should the nurse administer per dose? _____ Record your answer using one decimal place

ANS: 0.5

. Which type of seizure may be difficult to detect? a. Absence c. Simple partial b. Generalized d. Complex partial

ANS: A Absence seizures may go unrecognized because little change occurs in the childs behavior during the seizure. Generalized, simple partial, and complex partial seizures all have clinical manifestations that are observable.

When caring for the child with Reyes syndrome, the priority nursing intervention is to: a. Monitor intake and output. c. Observe for petechiae. b. Prevent skin breakdown. d. Do range-of-motion (ROM) exercises.

ANS: A Accurate and frequent monitoring of intake and output is essential for adjusting fluid volumes to prevent both dehydration and cerebral edema. Preventing skin breakdown, observing for petechiae, and doing ROM exercises are important interventions in the care of a critically ill or comatose child. Careful monitoring of intake and output is a priority

What effect does immobilization have on the cardiovascular system? a. Venous stasis b. Increased vasopressor mechanism c. Normal distribution of blood volume d. Increased efficiency of orthostatic neurovascular reflexes

ANS: A Because of decreased muscle contraction, the physiologic effects of immobilization include venous stasis. This can lead to pulmonary emboli or thrombi. A decreased vasopressor mechanism results in orthostatic hypotension, syncope, hypotension, decreased cerebral blood flow, and tachycardia. An altered distribution of blood volume is found, with decreased cardiac workload and exercise tolerance. Immobilization causes a decreased efficiency of orthostatic neurovascular reflexes, with an inability to adapt readily to the upright position and pooling of blood in the extremities in the upright position.

Which condition can result from the bone demineralization associated with immobility? a. Osteoporosis b. Urinary retention c. Pooling of blood d. Susceptibility to infection

ANS: A Bone demineralization leads to a negative calcium balance, osteoporosis, pathologic fractures, extraosseous bone formation, and renal calculi. Urinary retention is secondary to the effect of immobilization on the urinary tract. Pooling of blood is a result of the cardiovascular effects of immobilization. Susceptibility to infection can result from the effects of immobilization on the respiratory and renal systems.

. Which clinical manifestations would suggest hydrocephalus in a neonate? a. Bulging fontanel and dilated scalp veins b. Closed fontanel and high-pitched cry c. Constant low-pitched cry and restlessness d. Depressed fontanel and decreased blood pressure

ANS: A Bulging fontanel, dilated scalp veins, and separated sutures are clinical manifestations of hydrocephalus in neonates. Closed fontanel and high-pitched cry, constant low-pitched cry and restlessness, and depressed fontanel and decreased blood pressure are not clinical manifestations of hydrocephalus, but all should be referred for evaluation.

Latex allergy is suspected in a child with spina bifida. Appropriate nursing interventions include: a. Avoiding using any latex product. b. Using only nonallergenic latex products. c. Administering medication for long-term desensitization. d. Teaching the family about long-term management of asthma

ANS: A Care must be taken that individuals who are at high risk for latex allergies do not come in direct or secondary contact with products or equipment containing latex at any time during medical treatment. There are no nonallergenic latex products. At this time desensitization is not an option. The child does not have asthma. The parents must be taught about allergy and the risk of anaphylaxis

When assessing the child with osteogenesis imperfecta, the nurse should expect to observe: a. Discolored teeth. b. Below-normal intelligence. c. Increased muscle tone. d. Above-average stature

ANS: A Children with osteogenesis imperfecta have incomplete development of bones, teeth, ligaments, and sclerae. Teeth are discolored because of abnormal enamel. Despite their appearance, children with osteogenesis imperfecta have normal or above-normal intelligence. The child with osteogenesis imperfecta has weak muscles and decreased muscle tone. Because of compression fractures of the spine, the child appears short.

Which drug would be used to treat a child who has increased intracranial pressure (ICP) resulting from cerebral edema? a. Mannitol c. Atropine sulfate b. Epinephrine hydrochloride d. Sodium bicarbonate

ANS: A For increased ICP, mannitol, an osmotic diuretic, administered intravenously, is the drug used most frequently for rapid reduction. Epinephrine, atropine sulfate, and sodium bicarbonate are not used to decrease ICP.

An adolescent is scheduled for a leg amputation in 2 days for treatment of osteosarcoma. The nurses approach should include: a. Answering questions with straightforward honesty. b. Avoiding discussing the seriousness of the condition. c. Explaining that, although the amputation is difficult, it will cure the cancer. d. Assisting the adolescent in accepting the amputation as better than a long course of chemotherapy

ANS: A Honesty is essential to gain the cooperation and trust of the child. The diagnosis of cancer should not be disguised with falsehoods. The adolescent should be prepared in advance for the surgery so that there is time for reflection about the diagnosis and subsequent treatment. This allows questions to be answered. To accept the need for radical surgery, the child must be aware of the lack of alternatives for treatment. Amputation is necessary, but it will not guarantee a cure. Chemotherapy is an integral part of the therapy with surgery. The child should be informed of the need for chemotherapy and its side effects before surgery.

Which problem is most often associated with myelomeningocele? a. Hydrocephalus b. Craniosynostosis c. Biliary atresia d. Esophageal atresia

ANS: A Hydrocephalus is an associated anomaly in 80% to 90% of children. Craniosynostosis is the premature closing of the cranial sutures and is not associated with myelomeningocele. Biliary and esophageal atresias are not associated with myelomeningocele.

Spastic cerebral palsy is characterized by: a. Hypertonicity and poor control of posture, balance, and coordinated motion. b. Athetosis and dystonic movements. c. Wide-based gait and poor performance of rapid, repetitive movements. d. Tremors and lack of active movement.

ANS: A Hypertonicity and poor control of posture, balance, and coordinated motion are part of the classification of spastic cerebral palsy. Athetosis and dystonic movements are part of the classification of dyskinetic/athetoid cerebral palsy. Wide-based gait and poor performance of rapid, repetitive movements are part of the classification of ataxic cerebral palsy. Tremors and lack of active movement may indicate other neurologic disorders.

Which term is used to describe a type of fracture that does not produce a break in the skin? a. Simple b. Compound c. Complicated d. Comminuted

ANS: A If a fracture does not produce a break in the skin, it is called a simple or closed fracture. A compound or open fracture is one with an open wound through which the bone protrudes. A complicated fracture is one in which the bone fragments damage other organs or tissues. A comminuted fracture occurs when small fragments of bone are broken from the fractured shaft and lie in the surrounding tissue. These are rare in children

The nurse is preparing an adolescent with scoliosis for a Luque-rod segmental spinal instrumentation procedure. Which consideration should the nurse include? a. Nasogastric intubation and urinary catheter may be required. b. Ambulation will not be allowed for up to 3 months. c. Surgery eliminates the need for casting and bracing. d. Discomfort can be controlled with nonpharmacologic methods.

ANS: A Luque-rod segmental spinal instrumentation is a surgical procedure. Nasogastric intubation and urinary catheterization may be required. Ambulation is allowed as soon as possible. Depending on the instrumentation used, most patients walk by the second or third postoperative day. Casting and bracing are required postoperatively. The child usually has considerable pain for the first few days after surgery. Intravenous opioids should be administered on a regular basis

The most common problem of children born with a myelomeningocele is: a. Neurogenic bladder b. Intellectual impairment. c. Respiratory compromise. d. Cranioschisis

ANS: A Myelomeningocele is one of the most common causes of neuropathic (neurogenic) bladder dysfunction among children. Risk of intellectual impairment is minimized through early intervention and management of hydrocephalus. Respiratory compromise is not a common problem in myelomeningocele. Cranioschisis is a skull defect through which various tissues protrude. It is not associated with myelomeningocele.

. Which statement best describes a neuroblastoma? a. Diagnosis is usually made after metastasis occurs. b. Early diagnosis is usually possible because of the obvious clinical manifestations. c. It is the most common brain tumor in young children. d. It is the most common benign tumor in young children.

ANS: A Neuroblastoma is a silent tumor with few symptoms. In more than 70% of cases, diagnosis is made after metastasis occurs, with the first signs caused by involvement in the nonprimary site. In only 30% of cases is diagnosis made before metastasis. Neuroblastomas are the most common malignant extracranial solid tumors in children. The majority of tumors develop in the adrenal glands or the retroperitoneal sympathetic chain. They are not benign; they metastasize.

Osteosarcoma is the most common bone cancer in children. Where are most of the primary tumor sites? a. Femur b. Humerus c. Pelvis d. Tibia

ANS: A Osteosarcoma is the most frequently encountered malignant bone cancer in children. The peak incidence is Test Bank - Maternal Child Nursing Care by Perry (6th Edition, 2017) 817 between ages 10 and 25 years. More than half occur in the femur. After the femur, most of the remaining sites are the humerus, tibia, pelvis, jaw, and phalanges.

A young boy has just been diagnosed with pseudohypertrophic (Duchennes) muscular dystrophy. The management plan should include: a. Recommending genetic counseling. b. Explaining that the disease is easily treated. c. Suggesting ways to limit the use of muscles. d. Assisting the family in finding a nursing facility to provide his care.

ANS: A Pseudohypertrophic (Duchennes) muscular dystrophy is inherited as an X-linked recessive gene. Genetic counseling is recommended for parents, female siblings, maternal aunts, and their female offspring. No effective treatment exists at this time for childhood muscular dystrophy. Maintaining optimal function of all muscles for as long as possible is the primary goal. It has been found that children who remain as active as possible are able to avoid wheelchair confinement for a longer time. Assisting the family in finding a nursing facility is inappropriate at the time of diagnosis. When the child becomes increasingly incapacitated, the family may consider home-based care, a skilled nursing facility, or respite care to provide the necessary care.

The priority nursing intervention when a child is unconscious after a fall is to: a. Establish an adequate airway. b. Perform neurologic assessment. c. Monitor intercranial pressure. d. Determine whether a neck injury is present

ANS: A Respiratory effectiveness is the primary concern in the care of the unconscious child. Establishing an adequate airway is always the first priority. A neurologic assessment and determination of neck injury are performed after breathing and circulation are stabilized. Intracranial, not intercranial, pressure is monitored if indicated Test Bank - Maternal Child Nursing Care by Perry (6th Edition, 2017) 776 after airway, breathing, and circulation are maintained.

An adolescent boy is brought to the emergency department after a motorcycle accident. His respirations are deep, periodic, and gasping. There are extreme fluctuations in blood pressure. Pupils are dilated and fixed. What type of head injury should the nurse suspect? a. Brainstem c. Subdural hemorrhage Test Bank - Maternal Child Nursing Care by Perry (6th Edition, 2017) 778 b. Skull fracture d. Epidural hemorrhage

ANS: A Signs of brainstem injury include deep, rapid, periodic or intermittent, and gasping respirations. Wide fluctuations or noticeable slowing of the pulse, widening pulse pressure, or extreme fluctuations in blood pressure are consistent with a brainstem injury. Skull fracture and subdural and epidural hemorrhages are not consistent with these signs.

A child is upset because, when the cast is removed from her leg, the skin surface is caked with desquamated skin and sebaceous secretions. What should the nurse suggest to remove this material? a. Soak in a bathtub. b. Vigorously scrub the leg. c. Apply powder to absorb material. d. Carefully pick material off of the leg.

ANS: A Simple soaking in the bathtub is usually sufficient for the removal of the desquamated skin and sebaceous secretions. It may take several days to eliminate the accumulation completely. The parents and child should be advised not to scrub the leg vigorously or forcibly remove this material because it may cause excoriation and bleeding. Oil or lotion, but not powder, may provide comfort for the child.

A young girl has just injured her ankle at school. In addition to calling the childs parents, the most appropriate immediate action by the school nurse is to: a. Apply ice. b. Observe for edema and discoloration. c. Encourage child to assume a comfortable position. d. Obtain parental permission for administration of acetaminophen or aspirin.

ANS: A Soft-tissue injuries should be iced immediately. In addition to ice, the extremity should be rested, be elevated, and have compression applied. Observing for edema and discoloration, encouraging the child to assume a comfortable position, and obtaining parental permission or administration of acetaminophen or aspirin are not immediate priorities.

An important nursing intervention when caring for a child who is experiencing a seizure is to: a. Describe and record the seizure activity observed. b. Restrain the child when seizure occurs to prevent bodily harm. c. Place a tongue blade between the teeth if they become clenched. d. Suction the child during a seizure to prevent aspiration.

ANS: A Test Bank - Maternal Child Nursing Care by Perry (6th Edition, 2017) 785 When a child is having a seizure, the priority nursing care is observation of the child and seizure. The nurse then describes and records the seizure activity. The child should not be restrained, and nothing should be placed in his or her mouth. This may cause injury. To prevent aspiration, if possible, the child should be placed on his or her side, facilitating drainage

Four-year-old David is placed in Bucks extension traction for Legg-Calv-Perthes disease. He is crying with pain as the nurse assesses that the skin of his right foot is pale with an absence of pulse. What should the nurse do first? a. Notify the practitioner of the changes noted. b. Give the child medication to relieve the pain. c. Reposition the child and notify the physician. d. Chart the observations and check the extremity again in 15 minutes

ANS: A The absence of a pulse and change in color of the foot must be reported immediately for evaluation by the practitioner. Pain medication should be given after the practitioner is notified. This is an emergency condition; immediate reporting is indicated. The findings should be documented with ongoing assessment.

A toddler fell out of a second-story window. She had brief loss of consciousness and vomited four times. Since admission, she has been alert and oriented. Her mother asks why a computed tomography (CT) scan is required when she seems fine. The nurse should explain that the toddler: a. May have a brain injury. c. May start having seizures. b. Needs this because of her age. d. Probably has a skull fracture

ANS: A The childs history of the fall, brief loss of consciousness, and vomiting four times necessitate evaluation of a potential brain injury. The severity of a head injury may not be apparent on clinical examination but will be detectable on a CT scan. The need for the CT scan is related to the injury and symptoms, not the childs age, and is necessary to determine whether a brain injury has occurred

Which test is never performed on a child who is awake? a. Oculovestibular response b. Dolls head maneuver c. Funduscopic examination for papilledema d. Assessment of pyramidal tract lesions

ANS: A The oculovestibular response (caloric test) involves the instillation of ice water into the ear of a comatose child. The caloric test is painful and is never performed on a child who is awake or one who has a ruptured tympanic membrane. Dolls head maneuver, funduscopic examination, and assessment of pyramidal tract lesions can be performed on children who are awake

What is most descriptive of the therapeutic management of osteosarcoma? a. Treatment usually consists of surgery and chemotherapy. b. Amputation of the affected extremity is rarely necessary. c. Intensive irradiation is the primary treatment. d. Bone marrow transplantation offers the best chance of long-term survival.

ANS: A The optimal therapy for osteosarcoma is a combination of surgery and chemotherapy. Amputation is frequently required. Intensive irradiation and bone marrow transplantation are usually not part of the therapeutic management

Discharge planning for the child with juvenile arthritis includes the need for: a. Routine ophthalmologic examinations to assess for visual problems. b. A low-calorie diet to decrease or control weight in the less mobile child. c. Avoiding the use of aspirin to decrease gastric irritation. d. Immobilizing the painful joints, which are the result of the inflammatory process

ANS: A The systemic effects of juvenile arthritis can result in visual problems, making routine eye examinations important. Children with juvenile arthritis do not have problems with increased weight and often are anorexic and in need of high-calorie diets. Children with arthritis are often treated with aspirin. Children with arthritis are able to immobilize their own joints. Range-of-motion exercises are important for maintaining joint flexibility and preventing restricted movement in the affected joints

Which information should the nurse give to a child who is to have magnetic resonance imaging (MRI) of the brain? a. Your head will be restrained during the procedure. b. You will have to drink a special fluid before the test. c. You will have to lie flat after the test is finished. d. You will have electrodes placed on your head with glue

ANS: A To reduce fear and enhance cooperation during the MRI, the child should be made aware that the head will be restricted to obtain accurate information. Drinking fluids is usually done for neurologic procedures. A child should lie flat after a lumbar puncture, not after an MRI. Electrodes are attached to the head for an electroencephalogram


Conjuntos de estudio relacionados

Section 17 Unit 2 Contract Types and Their Effects

View Set

Personal Selling and Sales Management

View Set

aice us voacb quiz 3- reconstruction

View Set

Accounting - Chapter 4 - Survey of Accounting

View Set

Chapter 1 homework (study guide)

View Set

ch1: independent & dependent clauses: subordination & coordination: Quiz & chapter test

View Set